Цены снижены! Бесплатная доставка контурной маркировки по всей России

Формула угловой скорости через радиус: Формула угловой скорости

Содержание

Задача о точке, движущейся по окружности — Пространственное движение твердого тела

Здравствуйте! На второй неделе мы занимались тем, что определяли только положение твердого тела относительно неподвижного базиса. На практических занятиях на третьей неделе мы будем заниматься тем, что вычислять скорости и ускорения точек в твердом теле. На этом этапе у нас появляется понятие угловой скорости. Что нам нужно понимать? В кинематике точки у нас не было такого понятия, как «вращение», и для точки мы выводили только линейную скорость. В случае твердого тела у нас уже появляется возможность вращения. Что это значит? Что нам необходимо научиться определять вектор угловой скорости, который для твердого тела определяется единственным образом. И при помощи этого вектора угловой скорости мы будем учиться вычислять скорости и ускорения точек твердого тела, которые в общем случае совсем не обязаны совпадать друг с другом. Сделаем это на примере следующей задачи. Есть точка, которая движется по окружности радиуса r с постоянной скоростью v. Окружность радиуса r, точка движется со скоростью v, скорость направлена по касательной к траектории. И необходимо найти угловую скорость радиуса-вектора этой точки, то есть угловую скорость r. Вообще, радиус-вектор — это твердое тело в полном смысле этого слова, потому что в процессе движения расстояние между точками радиус-вектора никак не меняется. Поэтому понятие угловой скорости для радиус-вектора применить можно. Как мы будем искать угловую скорость? Мы знаем скорости двух точек у радиус-вектора. Мы знаем скорость начала — это центр окружности, скорость равна 0 — и знаем скорость конечной точки, точка A. Как мы их свяжем? Для того чтобы связать скорости этих двух точек, воспользуемся формулой Эйлера. Она говорит, что скорость точки A можно вычислить при помощи скорости точки O следующим образом: добавив векторное произведение вектора угловой скорости на вектор OA. Чтобы пользоваться этой формулой, давайте введем вспомогательную систему координат, на которую и будем проецировать. Ось z — вертикально вверх, на нас, ось y — параллельно вектору скорости, ось x — вдоль радиус-вектора так, чтобы составлять правую тройку. Скорость точки A в этой системе координат записывается как [0, v, 0]. Скорость точки O равна 0, потому что это центр окружности. И осталось вычислить векторное произведение. У вектора угловой скорости будем считать, что все три компоненты неизвестны, то есть [ωx, ωy, ωz]. И векторно умножим на вектор OA. Вектор OA в этой системе координат имеет координаты [R, 0, 0]. Давайте вычислим векторное произведение. Первый элемент равен 0, второй элемент равен ωz * R, и третий элемент равен −ωy * R. Если сравнить правую и левую части выражения, то что мы видим? Что компонента угловой скорости по z равна v / R. Компонента угловой скорости по оси y равна 0. И мы ничего не можем сказать про компоненту угловой скорости по оси x. Почему так получилось? Как это можно объяснить на пальцах? На самом деле, действительно, радиус-вектор — это отрезок, это вырожденное твердое тело, мы ничего не знаем про его повороты вокруг его длины. Если мы знаем скорости двух точек, то они нам никак не ответят на вопрос, вращается ли как-то стержень, отрезок вокруг своей оси, длины. Как это можно показать другим образом, математически? Что у нас есть? У нас есть формула Эйлера, которая говорит, что если скорости двух точек известны, в смысле ну да, если известны скорости двух точек, то вектор угловой скорости мы можем попытаться определить из формулы Эйлера. Давайте выпишем через матрицу угловой скорости или через матрицу радиус-векторов. Если у нас есть скорости двух точек, то разница между ними по формуле Эйлера — это следующая вещь. Мы можем переписать это через кососимметричную матрицу для угловой скорости. Матрица угловой скорости будет выглядеть: −ωz, ωy, −ωx. И так как она кососимметричная, то на нижней диагонали те же элементы со знаком «−», то есть ωz, −ωy, ωx. И эту матрицу нужно умножить на вектор OA, то есть вектор с компонентами [rx, ry, rz]. Ну а в таком виде рассуждать не очень удобно, давайте перепишем чуть-чуть иначе. Вместо кососимметричной матрицы для угловой скорости сделаем кососимметричную матрицу для радиус-векторов. Можно проверить непосредственным вычислением и убедиться в том, что выражение выше полностью совпадает по результату с выражением, которое я сейчас выпишу: компонента радиус-вектора по оси z, минус компонента радиус-вектора по оси y, компонента радиус-вектора по оси x, здесь все так же и остается. Матрица кососимметричная, значит, следующие элементы со знаком «−». И если мы умножим это на компоненты вектора, на вектор угловой скорости — [ωx, ωy, ωz], то это будет точно так же равняться разности скоростей двух точек. То есть на самом деле это у нас алгебраическое уравнение, которое говорит, что вектор, состоящий из разницы скоростей — это некая матрица A умножить на вектор угловых скоростей. Что мы из этого выражения хотим найти? Мы хотим по известным разницам скоростей и по известной матрице A научиться вычислять вектор угловой скорости. По идее кажется, что ничего сложного, да? Три компоненты, три уравнения, три неизвестных — все должно определяться. Но если вспомнить теорию линейных уравнений, теорию линейных систем, то вы знаете, что если матрица A — вырожденная, то есть если детерминант матрицы A = 0, то у такой системы может быть бесконечно много решений либо вообще не существовать решений. Давайте в нашем случае посмотрим, чему равен детерминант. Можно непосредственным вычислением проверить и увидеть, что на самом деле он реально 0, то есть можно даже не выписывать. Вот. Что это значит? Что из теории алгебраических систем, так как матрица вырождена, то двух точек, двух скоростей нам недостаточно для того, чтобы определить все три компоненты вектора угловой скорости, что мы здесь, в принципе, и получили. Нам не хватило компонент. И на самом деле, здесь мы не привязывались к тому, что у нас только отрезок — это сделано в общем случае. Если у нас только две точки, то мы не сможем однозначно определить вектор угловой скорости, нам необходима третья точка с известной скоростью. Тогда мы сможем разрешить систему. Так, давайте для нашей задачи покажем, что если действительно взять еще какую-то третью точку, то угловая скорость определится. Что сделаем? Давайте дополнительно введем еще в точке A триэдр. То есть введем систему координат z’x’y’ и потребуем от этой системы координат, чтобы ось z’ всегда оставалась при движении параллельна сама себе, то есть чтобы скорости точек с координатами (0, 0, a) для любых a были равны скорости точки A. Скорость точки A, как и раньше, по касательной к траектории, она известна, она равна v. Даст ли нам это что-то новое, для того чтобы определить угловую скорость? Давайте проверим. Скорость любой точки на триэдре на оси z’ — обозначу ее таким вот образом — может быть вычислена из точки O, из скорости точки O, опять же по такой формуле, по формуле Эйлера. Что это скорость точки O плюс векторное произведение вектора угловой скорости умножить на вектор O, давайте я обозначу, A’. То есть A’ — это точка на оси z’ над точкой A. Как и раньше, скорость точки O у нас 0. Компоненты вектора ω мы определили каким образом? Что мы не знаем чему равно ωx, знаем, что ωy = 0, а ωz = v / R. И умножим векторно на вектор OA’. По оси x’ компонента этого вектора равна R, по оси y равна 0, и по z равна произвольному значению a. Вычислим векторное произведение. Первый элемент — 0, второй элемент — −ωx * a + v, и третий элемент — 0. Причем что мы требовали? Чтобы точка A’ на триэдре имела ту же самую скорость, что наша исходная точка на окружности, то есть [0, v, 0]. А отсюда теперь уже однозначно получаем, что третья компонента угловой скорости = 0. То есть что мы показали? Что трех точек достаточно для того, чтобы найти угловую скорость — трех точек, не лежащих на одной прямой, простите, достаточно, чтобы найти угловую скорость. И что мы сделали еще, на самом деле, когда ввели такой триэдр? Потребовав, что ось z’ остается все время параллельна сама себе, мы запретили на самом деле радиус-вектору поворачиваться вокруг своей оси. Вот так. Задача решена, спасибо.

Вращательное движение

Страница 1 из 3

Существует большое количество расчетных задач, которые моделируют явления, происходящие в различных вращающихся агрегатах или около них. При постановке подобной численной задачи важно выбрать способ описания вращения в численной модели, который будет корректен с точки зрения физики и оптимален с точки зрения производительности вычислений. FlowVision позволяет задавать вращение различными способами: с помощью вращающейся локальной системы координат; с помощью подвижных тел; с помощью скользящих поверхностей. С целью помочь пользователю разобраться с постановкой такого типа задач, рассмотрены примеры задач разного типа, начиная с физико-математических основ.                                                                                                          

1. Кинематика вращательного движения

1.1. Вращательное движение материальной точки

Вращательное движение материальной точки (м.т.) вокруг неподвижной оси – это движение материальной точки по окружности радиуса R, центр которой лежит на неподвижной относительно данной системы отсчета прямой (ось вращения), перпендикулярной плоскости, в которой лежит траектория точки.

Рис.1.

Вращательное движение тела вокруг неподвижной оси — движение тела, при котором все его точки, двигаясь в параллельных плоскостях, описывают окружности с центрами, лежащими на одной неподвижной прямой, называемой осью вращения. Тело, совершающее вращательное движение, имеет одну степень свободы, и его положение относительно данной системы отсчёта определяется углом поворота φ между неподвижной полуплоскостью и полуплоскостью, жёстко связанной с телом, проведёнными через ось вращения.

Рис.2.

1.2. Угол поворота

Угол φ считается положительным, если он отложен от неподвижной плоскости в направлении против хода часовой стрелки (для наблюдателя, смотрящего с положительного конца оси Az), и отрицательным, если по ходу часовой стрелки. Чтобы знать положение в любой момент времени, надо знать зависимость угла φ от времени t, т.е. φ=f(t).          

1.3. Основные кинематические характеристики вращательного движения

Основными кинематическими характеристиками вращательного движения являются угловая скорость   и угловое ускорение  .                                        
Угловая скорость и угловое ускорение величины векторные. Вектор угловой скорости направлен вдоль оси вращения в ту сторону, откуда вращение видно происходящим против хода часовой стрелки (рис.3). Такой вектор определяет сразу и модуль угловой скорости, и ось вращения, и направление вращения вокруг этой оси. Аналогично углу поворота, когда вращение происходит против хода часовой стрелки (для наблюдателя, смотрящего с положительного конца оси Az) ω>0, а когда по ходу часовой стрелки, то ω<0. Таким образом, знак ωопределяет направление вращения.

а)    б)    в)

Рис.3

1.4. Прочие кинематические характеристики

Скорость точки M на расстоянии R от оси (рис.2):  

Тангенциальная составляющая ускорения точки M (рис.3б): 

Нормальная составляющая ускорения точки M (рис.3б): 

Полное ускорение точки M (рис.3б): 

Формула Эйлера (рис.3в):  

2. Силы инерции, действующие на материальную точку во вращающейся системе отсчета

2.1. Материальная точка, покоящаяся во вращающейся системе отсчета

Если рассмотреть движение вращающейся точки M, то относительно  неподвижной системы координат (СК) XYZ (рис.4а) силу, действующую на неё можно определить  из второго закона Ньютона:  . Относительно вращающейся системы координат X’Y’Z’ точка M неподвижна (рис.4б). Это обеспечивается тем, что равнодействующая сил уравновешивается инерциальной силой (центробежной):  .

Рис.4 (а,б)

2.2. Материальная точка, движущаяся во вращающейся системе отсчета

Если же точка движется во вращающейся системе отсчета, то помимо центробежной силы на неё действует ещё одна сила инерции – сила Кориолиса   (рис.5). Направление силы Кориолиса определяется правилом правого винта.   


Рис. 5.

Таким образом, при переходе от основной неподвижной СК к локальной СК, которая является вращающейся системой отсчета, появляются дополнительные составляющие вектора силы, которые действуют на материальную точку: центробежная сила    и сила Кориолиса   .

Движение по окружности

Движение по окружности — простейший случай криволинейного движения тела. Когда тело движется вокруг некоторой точки, наряду с вектором перемещения удобно ввести угловое перемещение ∆φ (угол поворота относительно центра окружности), измеряемое в радианах. 

Зная угловое перемещение, можно вычислить длину дуги окружности (путь), которую прошло тело. 

∆l=R∆φ

Если угол поворота мал, то ∆l≈∆s.

Проиллюстрируем сказанное:

Угловая скорость

При криволинейном движении вводится понятие угловой скорости ω, то есть скорости изменения угла поворота. 

Определение. Угловая скорость

Угловая скорость в данной точке траектории — предел отношения углового перемещения ∆φ к промежутку времени ∆t, за которое оно произошло. ∆t→0.

ω=∆φ∆t, ∆t→0.

Единица измерения угловой скорости — радиан в секунду (радс).

Существует связь между угловой и линейной скоростями тела при движении по окружности. Формула для нахождения угловой скорости:

ω=vR

Нормальное ускорение

При равномерном движении по окружности, скорости v и ω остаются неизменными. Меняется только направление вектора линейной скорости.

При этом равномерное движение по окружности на тело действует центростремительное, или нормальное ускорение, направленное по радиусу окружности к ее центру. 

an=∆v→∆t, ∆t→0

Модуль центростремительного ускорения можно вычислить по формуле:

an=v2R=ω2R

Докажем эти соотношения.

Рассмотрим, как изменяется вектор v→ за малый промежуток времени ∆t. ∆v→=vB→-vA→.

В точках А и В вектор скорости направлен по касательной к окружности, при этом модули скоростей в обеих точках одинаковы.

По определению ускорения:

a→=∆v→∆t, ∆t→0

Взглянем на рисунок:

Треугольники OAB и BCD подобны. Из этого следует, что OAAB=BCCD.

Нужна помощь преподавателя?

Опиши задание — и наши эксперты тебе помогут!

Описать задание

Если значение угла ∆φ мало, расстояние AB=∆s≈v·∆t. Принимая во внимание, что OA=R и CD=∆v для рассмотренных выше подобных треугольников получим:

Rv∆t=v∆v или ∆v∆t=v2R

При ∆φ→0, направление вектора ∆v→=vB→-vA→ приближается к направлению на центр окружности. Принимая, что ∆t→0, получаем:

a→=an→=∆v→∆t; ∆t→0; an→=v2R.

При равномерном движении по окружности модуль ускорения остается постоянным, а направление вектора изменяется со временем, сохраняя ориентацию на центр окружности. Именно поэтому это ускорение называется центростремительным: вектор в любой момент времени направлен к центру окружности. 

Запись центростремительного ускорения в векторной форме выглядит следующим образом:

an→=-ω2R→.

Здесь R→ — радиус вектор точки на окружности с началом в ее центре.

Тангенциальное ускорение

В общем случае ускорение при движении по окружности состоит из двух компонентов — нормальное, и тангенциальное.

Рассмотрим случай, когда тело движется по окружности неравномерно. Введем понятие тангенциального (касательного) ускорения. Его направление совпадает с направлением линейной скорости тела и в каждой точке окружности направлено по касательной к ней.

aτ=∆vτ∆t; ∆t→0

Здесь ∆vτ=v2-v1  — изменение модуля скорости за промежуток ∆t

Направление полного ускорения определяется векторной суммой нормального и тангенциального ускорений.

Движение по окружности в плоскости можно описывать при помощи двух координат: x и y. В каждый момент времени скорость тела можно разложить на составляющие vx и vy.

Если движение равномерное, величины vx и vy а также соответствующие координаты будут изменяться во времени по гармоническому закону с периодом T=2πRv=2πω

 

Глава 7. Движемся по орбитам – FIZI4KA

В этой главе. . .

  • Постигаем равномерное вращательное движение
  • Изучаем угловое ускорение
  • Испытываем влияние центростремительной силы
  • Учитываем перемещение, скорость и ускорение
  • Движемся по орбите под действием законов Ньютона и силы гравитационного притяжения
  • Поддерживаем вращение в вертикальной плоскости

Вращательное движение выполняют искусственные спутники вокруг планет, гоночные автомобили по трекам и даже пчелы вокруг ульев. В предыдущих разделах рассматривались такие характеристики прямолинейного движения, как перемещение, скорость и ускорение. В этой главе мы снова рассмотрим их, но теперь уже для вращательного движения.

Для перечисленных выше характеристик прямолинейного движения есть аналоги, характеризующие вращательное движение, а именно: угловое перемещение, угловая скорость и угловое ускорение. Как видно из их названия, роль перемещения во вращательном движении играет угол. Угловая скорость обозначает величину угла поворота за единицу времени, а угловое ускорение — изменение угловой скорости за единицу времени. Все, что нужно сделать, чтобы освоить премудрости вращательного движения, это взять уравнения прямолинейного движения и заменить в них одни характеристики другими: перемещение поменять на угол, скорость — на угловую скорость и ускорение — на угловое ускорение.

Держим курс: равномерное вращательное движение

Если объект движется с постоянной по величине скоростью по окружности, то такое движение называется равномерным вращательным движением. Примерами такого движения являются движение гоночного автомобиля по круглому треку и стрелки на циферблате часов. На рис. 7.1 показан мяч для игры в гольф, привязанный нитью к шесту и совершающий движение по окружности. Мяч совершает движение с одинаковой по величине скоростью, но с изменяющимся направлением. Потому такое движение мяча называется равномерным вращательным движением.

Время, которое требуется мячику (или какому-либо другому объекту), чтобы полностью обогнуть окружность, называется периодом и обозначается символом ​\( T \)​. Период и линейную скорость можно легко связать, если известно пройденное расстояние, т.е. длина окружности ​\( 2\pi r \)​, а точнее ее радиус ​\( r \)​. Итак, линейная скорость мячика ​\( v \)​ равна:

а период вращения ​\( T \)​ равен:

Допустим, что длина нити равна 1 м, а период вращения равен 0,5 с. Чему в таком случае будет равна линейная скорость мячика? Подставим численные значения в одно из предыдущих соотношений и получим:

Итак, мячик вращается с линейной скоростью 13 м/с!

Меняем направление: центростремительное ускорение

При вращательном движении по окружности линейная скорость мячика постоянно меняет направление, как показано на рис. 7.2. Ускорение, характеризующее такое изменение скорости, называется центростремительным (или центробежным). В любой точке вращательного движения с постоянной величиной и меняющимся направлением вектор линейной скорости перпендикулярен радиусу.

Это правило справедливо для всех объектов: вектор линейной скорости объекта, равномерно вращающегося по окружности, всегда перпендикулярен радиусу окружности.

Если в показанных на рис. 7.2 положениях нить, удерживающая мяч, оборвется, то куда полетит мяч? Если в этот момент вектор линейной скорости направлен влево, то мяч полетит влево, а если этот вектор направлен вправо, то мяч полетит вправо, и т.д. Этот, казалось бы, простой и интуитивно понятный момент часто вызывает трудности у тех, кто впервые постигает физику.

Всегда следует помнить, что вектор линейной скорости объекта, выполняющего равномерное вращательное движение, всегда направлен под прямым углом к радиусу вращения в текущей точке траектории. (В общем случае неравномерного криволинейного движения эта компонента вектора скорости, перпендикулярная радиусу вращения и касательная к траектории движения, называется тангенциальной компонентой, а перпендикулярная ей компонента — нормальной компонентой. — Примеч. ред.)

Управляем скоростью с помощью центростремительного ускорения

Особенностью равномерного вращательного движения является постоянство величины линейной скорости. Это значит, что вектор ускорения не имеет компоненты, параллельной вектору линейной скорости, поскольку в противном случае величина линейной скорости менялась бы. Однако при равномерном вращательном движении меняется только направление линейной скорости. Такое изменение линейной скорости поддерживается центростремительным ускорением, направленным к центру окружности вращения и перпендикулярно вектору линейной скорости.

В примерах на рис. 7.1 и 7.2 на мяч со стороны нити действует сила натяжения нити, которая поддерживает его движение по окружности. Именно эта сила сообщает мячу центростремительное ускорение ​\( a_ц \)​, вектор которого показан на рис. 7.1. (Попробуйте раскрутить мяч с помощью привязанной к нему нити, и вы сразу же почувствуете действие этой силы со стороны нити.)

Часто возникает вопрос: если вектор ускорения мяча направлен к центру окружности, то почему мяч не движется к центру? Дело в том, что при равномерном вращательном движении это ускорение меняет только направление, а не величину линейной скорости.

Определяем величину центростремительного ускорения

Нам уже известно направление вектора центростремительного ускорения, а чему же равна его величина? Итак, величина центростремительного ускорения объекта, равномерно движущегося с линейной скоростью ​\( v \)​ по окружности с радиусом ​\( r \)​, равна:

Как видите, величина центростремительного ускорения обратно пропорциональна радиусу окружности ​\( r \)​ и прямо пропорциональна квадрату скорости ​\( v \)​. Поэтому не удивительно, что автомобиль на более крутых поворотах испытывает более сильное центростремительное ускорение.

Стремимся к центру: центростремительная сила

На крутых поворотах действие центростремительного ускорения обеспечивается трением шин по дороге. Какую силу нужно приложить, чтобы удержать движущийся со скоростью ​\( v \)​ автомобиль на повороте с радиусом кривизны ​\( r \)​?

Допустим, что в примере на рис. 7.1 легкий мяч заменили на тяжелое пушечное ядро. Теперь, чтобы поддерживать движение ядра по окружности с тем же радиусом и периодом вращения, потребуется гораздо большая сила.

Дело в том, что сила ​\( F=ma \)​ равна произведению ускорения ​\( a \)​ и массы ​\( m \)​, а значит, увеличение массы объекта (замена мяча на ядро) неизбежно приводит к необходимости увеличения силы для обеспечения прежнего ускорения.

Центростремительная сила ​\( F_ц \)​, необходимая для равномерного вращения по окружности с радиусом ​\( r \)​ объекта массой ​\( m \)​ с постоянной скоростью ​\( v \)​, равна:

С помощью этого уравнения можно легко определить силу, необходимую для равномерного вращения объекта по окружности с известной массой, скоростью и радиусом окружности.

Обратите внимание, что если объект движется по той же окружности, но с разной скоростью, то он будет испытывать разную центростремительную силу.

В примерах на рис. 7.1 и 7.2 мяч движется со скоростью ​\( v \)​ = 13 м/с и удерживается нитью длиной 1,0 м, т.е. в данном случае радиус окружности ​\( r \)​ = 1 м. Какая сила потребуется, чтобы поддерживать такое же движение для пушечного ядра с массой 10 кг? Подставляя численные значения в уже известную нам формулу, получим:

Приличная сила! Остается только надеяться, что ваши руки достаточно сильны, чтобы удержать ядро.

Является ли центростремительная сила реальной силой?

Центростремительная сила не является каким-то особым типом взаимодействия. Она имеет отношение только к объекту, движущемуся по криволинейной траектории, и необходима для удержания объекта на данной траектории. Поэтому ее часто называют центростремительно-необходимой силой. Довольно часто новички считают центростремительную силу каким-то новым фундаментальным типом взаимодействия. И это понятно, поскольку известные нам силы (например, сила гравитации и сила трения) имеют вполне определенный источник, который не зависит от траектории движения. Но это совсем не так для центростремительной силы. Центростремительная сила возникает из необходимости удержания объекта на криволинейной траектории. Сумма всех остальных сил, действующих на объект, который движется по криволинейной траектории, должна быть равна центростремительной силе. (Если объект движется по прямолинейной траектории, а затем ему нужно изменить направление движения, то для этого придется приложить силу, равную центростремительной силе. — Примеч. ред.)

Вписываемся в повороты: учитываем радиус и наклон

Если вам приходилось ехать на автомобиле или велосипеде или даже бежать трусцой, то наверняка вы заметили, что в крутой поворот проще вписаться, если поверхность дороги немного наклонена внутрь поворота. Из опыта известно, что чем больше наклон, тем проще вписаться в поворот. Это объясняется тем, что в таком случае на вас действует меньшая центростремительная сила. Центростремительная сила обеспечивается силой трения о поверхность дороги. Если поверхность дороги покрыта льдом, то сила трения становится меньше и потому часто не удается вписаться в поворот на обледеневшей дороге на большой скорости.

Представьте, что автомобилю с массой 1000 кг нужно вписаться в поворот с радиусом Юм, а коэффициент трения покоя (подробнее о нем см. главу6) равен 0,8. (Здесь используется коэффициент трения покоя, поскольку предполагается, что шины по поверхности дороги.) Какую максимальную скорость может развить этот автомобиль без риска не вписаться в поворот. Итак, сила трения покоя шин о поверхность дороги ​\( F_{трение\,покоя} \)​ должна обеспечивать центростремительную силу:

где ​\( m \)​ — это масса автомобиля, ​\( v \)​ — его скорость, ​\( r \)​ — радиус, ​\( \mu_п \)​ — коэффициент трения покоя, a ​\( g \)​ = 9,8 м/с2 — ускорение свободного падения под действием силы гравитации. Отсюда легко находим скорость:

(Обратите внимание, что максимальная безопасная скорость прохождения поворота не зависит от массы автомобиля. — Примеч. ред.)

Это выражение выглядит очень просто, а после подстановки в него численных значений получим:

Итак, максимальная скорость безопасного проезда при таком повороте равна 8,9 м/с. Пересчитаем в единицы “км/ч”, в которых скорость указана на спидометре, и сравним. Получается, что 8,9 м/с = 32 км/ч, а на спидометре всего 29 км/ч. Прекрасно, но далеко не все водители умеют так быстро рассчитывать безопасную скорость прохождения поворотов. Поэтому конструкторы дорог часто строят повороты с наклоном внутрь, чтобы обеспечить центростремительное ускорение не только за счет силы трения, но и за счет горизонтальной компоненты силы гравитации.

На рис. 7.3 показан пример поворота дороги с некоторым наклоном под углом ​\( \theta \)​ к горизонтали. Предположим, что конструкторы решили полностью обеспечить центростремительное ускорение только за счет горизонтальной компоненты силы гравитации (т.е. без учета силы трения) ​\( F_н\sin\theta \)​, где ​\( F_н \)​ — это нормальная сила (подробнее о ней см. в главе 6). Тогда:

В вертикальном направлении на автомобиль действует сила гравитации ​\( mg \)​, которая уравновешивается вертикальной компонентой нормальной силы \( F_н\cos\theta \):

или, иначе выражая это соотношение, получим:

Подставляя это выражение в прежнее соотношение между центростремительной силой и нормальной силой, получим:

Поскольку ​\( \sin\theta/\!\cos\theta=tg\,\theta \)​ в то

Отсюда легко получаем, что угол наклона поворота дороги ​\( \theta \)​ равен:

Именно это уравнение используют инженеры при проектировании дорог. Обратите внимание, что масса автомобиля не влияет на величину угла, при котором центростремительная сила полностью обеспечивается только горизонтальной компонентой нормальной силы. Попробуем теперь определить величину угла наклона поворота с радиусом 200 м для автомобиля, движущегося со скоростью 100 км/ч или 27,8 м/с:

Для обеспечения безопасного движения автомобиля со скоростью 100 км/ч в повороте с радиусом 200 м без учета силы трения, инженеры должны создать наклон около 22°. Отлично, из вас может получиться неплохой инженер-конструктор автомагистралей!

Вращательное движение: перемещение, скорость и ускорение

Если вы привыкли решать задачи о прямолинейном движении типа “некто движется из пункта А в пункт Б”, то задачи о вращательном движении можно формулировать аналогично, но для этого нужно приобрести некоторый опыт. На рис. 7.1 мяч движется криволинейно по окружности, а не прямолинейно по линии. Это движение можно было бы описать как комбинацию прямолинейных движений с координатами X и Y. Однако гораздо удобнее характеризовать его иначе, а именно как вращательное движение с одной координатой ​\( \theta \)​. В данном примере вращательного движения перемещение можно характеризовать углом \( \theta \) так же, как в прямолинейном движении перемещение характеризуется расстоянием \( s \). (Более подробно перемещение при прямолинейном движении описывается в главе 3.)

Стандартной единицей измерения перемещения при вращательном движении является радиан (рад), а не градус. Полная окружность охватывает угол величиной ​\( 2\pi \)​ радиан, что равно 360°. Соответственно, половина окружности охватывает угол величиной ​\( \pi \)​ радиан, а четверть окружности — ​\( \pi/2 \)​.

Как преобразуются величины углов из градусов в радианы и обратно? Достаточно определить, сколько радиан приходится на один градус, т.е. вычислить отношение ​\( 2\pi \)​/360°. Например, величина угла 45° в радианах равна:

Аналогично, для преобразования величины угла из радианов в градусы следует определить, сколько градусов приходится на один радиан, т.е. вычислить отношение 360°/​\( 2\pi \)​. Например, величина угла ​\( \pi/2 \)​ в градусах равна:

Формулировка вращательного движения в терминах прямолинейного движения очень удобна. Напомним основные формулы прямолинейного движения, которые подробно описываются в главе 3:

Теперь для вывода аналогичных основных формул вращательного движения достаточно в формулах прямолинейного движения вместо расстояния ​\( s \)​, которое характеризует прямолинейное перемещение, подставить угол ​\( \theta \)​, который характеризует угловое перемещение. А как определяется угловая скорость? Очень просто. Угловая скорость ​\( \omega \)​ определяется аналогично, как изменение угла за единицу времени, и равна количеству радианов, пройденных за секунду:

Обратите внимание, как похоже это выражение для угловой скорости на выражение для линейной скорости:

Давайте теперь вычислим угловую скорость мяча на рис. 7.1. Он совершает полный круг, охватывающий ​\( 2\pi \)​ радиан, за 1/2 с, а значит, его угловая скорость равна:

(Величина угла, выраженная в радианах, равна отношению длины дуги окружности к длине ее радиуса. Поэтому радиан — это безразмерная величина, и ее обозначение (рад) часто опускается. Соответственно, угловую скорость принято указывать “в обратных секундах” как с-1, т.е. без указания единицы измерения углов. — Примеч. ред.)

Угловое ускорение ​\( \alpha \)​ определяется аналогично линейному ускорению:

Оно определяется как изменение угловой скорости за единицу времени и измеряется в радианах на секунду в квадрате.{-1} \), то чему равно угловое ускорение? Подставим эти численные значения в предыдущую формулу и получим:

Итак, для описания вращательного движения у нас есть следующие аналоги: для линейного перемещения ​\( s \)​ — угловое перемещение ​\( \theta \)​, для линейной скорости ​\( v \)​ — угловая скорость ​\( \omega \)​ и для линейного ускорения ​\( a \)​ — угловое ускорение ​\( \alpha \)​.

На основании этой аналогии можно легко вывести основные формулы вращательного движения (подобно основным формулам прямолинейного движения, которые подробно описываются в главе 3):

Более подробно эти выражения рассматриваются далее в главе 10 при описании момента импульса и момента силы.

Бросаем яблоко: закон всемирного тяготения Ньютона

Чтобы проводить опыты с вращательным движением, необязательно привязывать мячики к нитям и вращать их вокруг себя. Например, Луне совсем не нужны никакие нити, чтобы вращаться вокруг Земли. А дело в том, что необходимую центростремительную силу, вместо силы натяжения нити, обеспечивает сила гравитационного притяжения.

Один из важнейших законов физики, а именно закон всемирного тяготения, вывел еще сэр Исаак Ньютон. Согласно этому закону любые два тела притягиваются друг к другу с некоторой силой. Величина этой силы притяжения между телами с массами ​\( m_1 \)​ и ​\( m_2 \)​, которые находятся на расстоянии ​\( r \)​ друг от друга, равна:

где ​\( G \)​ — это константа, равная 6,67·10-11 Н·м2/кг2.

Благодаря этому уравнению можно легко вычислить силу гравитационного притяжения между двумя телами. Например, какова сила гравитационного притяжения между Землей и Солнцем? Солнце имеет массу около 1,99·1030 кг, Земля — 5,97·1024 кг, а расстояние между ними равно 1,50·1011 м. Подставляя эти числа в закон всемирного тяготения Ньютона, получим:

Историческая яблоня

Как известно, яблоко упало на голову Исаака Ньютона, и он открыл закон всемирного тяготения. Неужели это так и было? Правда ли, что какое-то падающее яблоко натолкнуло его на верную мысль или, по крайней мере, привлекло внимание Ньютона к данной теме? Согласно последним историческим исследованиям, весьма маловероятно, что именно падение яблока на голову великого ученого вдохновило его.2 \)​ для силы гравитационного притяжения справедливо независимо от расстояния между двумя массивными телами. В обыденных ситуациях часто приходится иметь дело с небольшими (по сравнению с размерами Земли) объектами на поверхности Земли, т.е. на фиксированном расстоянии между центром Земли и центром небольшого объекта. Силу гравитационного притяжения (или силу тяжести), действующую на небольшой объект, часто называют весом. Вес ​\( F_g \)​ равен произведению массы ​\( m \)​ на ускорение свободного падения ​\( g \)​, т.е. ​\( F_g = mg \)​. Массу измеряют в граммах, килограммах, центнерах, каратах и т.д., а вес — в динах, ньютонах и даже фунт-силах.

Попробуем вычислить ускорение свободного падения на поверхности Земли, пользуясь законом всемирного тяготения. Формула веса тела с массой ​\( m_1 \)​ нам известна:

Она создается силой гравитационного притяжения между этим телом и Землей и равна этой силе:

Здесь ​\( r \)​ — это радиус Земли, равный 6,38·106 м, а ​\( m_2 \)​ — ее масса, равная 5,97·1024 кг.

Сокращая массу тела ​\( m_1 \)​ в обеих половинах предыдущего равенства, получим:

Подставляя численные значения, получим:

Так, благодаря закону всемирного тяготения Ньютона мы смогли вычислить значение ускорения свободного падения, уже известное нам из прежних глав. Как видите, для этого нам потребовались значения константы всемирного тяготения ​\( G \)​, радиуса Земли ​\( r \)​ и ее массы ​\( m_2 \)​. (Конечно, значение ускорения свободного падения ​\( g \)​ можно определить экспериментально, измеряя время падения предмета с известной высоты. Но, согласитесь, гораздо интересней использовать последнюю формулу, для применения которой потребуется экспериментально измерить… радиус и массу Земли. Шутка!)

Исследуем орбитальное движение с помощью закона всемирного тяготения

Небесные тела в космическом пространстве из-за силы гравитационного притяжения вращаются друг относительно друга: спутники — вокруг своих планет (как Луна — вокруг Земли), планеты — вокруг звезд (как Земля — вокруг Солнца в Солнечной системе), а звезды — вокруг центра Галактики (как Солнце — вокруг центра нашей галактики, т.е. Млечного пути), а Галактика — вокруг местной группы галактик (как Млечный путь — вокруг нашей Местной группы галактик). Во всех этих случаях тела удерживаются центростремительной силой, которую обеспечивает сила гравитации. Как показано ниже, такая центростремительная сила несколько отличается от той, которая известна нам по прежнему примеру с вращающимся на нитке мячом для игры в гольф. В следующих разделах рассматриваются широко известные законы вращения тел под действием силы гравитационного притяжения, так называемые законы Кеплера, т.е. соотношения между параметрами вращательного движения: периодами вращения, радиусами и площадями орбит вращения.

Вычисляем скорость спутника

Чему равна скорость спутника, вращающегося вокруг планеты по орбите с постоянным радиусом? Ее можно легко определить, приравнивая центростремительную силу:

и силу гравитации:

В итоге получаем:

После простых алгебраических операций получим следующее выражение для скорости вращения:

Это уравнение определяет скорость вращения спутника по постоянной орбите независимо от его происхождения, будь-то искусственный спутник Земли, как рукотворный космический корабль на постоянной орбите, или естественный спутник Земли, как Луна.

Подсчитаем скорость вращения искусственного спутника Земли, вращающегося вокруг Земли. Для этого нужно в предыдущую формулу подставить массу Земли и расстояние от космического орбитального спутника до центра Земли.

Рукотворные спутники Земли обычно вращаются на высоте около 640 км, а радиус Земли, как известно, равен 6,38·106 м. Можно считать, что искусственные спутники вращаются на круговой орбите с радиусом около 7,02·106 м. Подставляя это и другие известные нам численные значения в предыдущую формулу, получим:

В этом месте нужно сделать несколько важных замечаний.

Значение 7,02·106 м в знаменателе обозначает расстояние от спутника до центра Земли, а не расстояние от спутника до поверхности Земли, равное 640 км. Помните, что в законе всемирного тяготения под расстоянием между телами подразумевается расстояние между их центрами масс, а не между их поверхностями.

В данном примере предполагается, что космический корабль находится достаточно высоко и не испытывает влияние атмосферы, например силу трения от соприкосновения с ней. На самом деле это не так. Даже на такой большой высоте как 640 км, космический корабль теряет скорость, вследствие трения в разреженных слоях атмосферы. В результате его скорость уменьшается, а сам корабль постепенно снижается. (Более подробно об этом рассказывается ниже.)

Движение искусственного спутника вокруг Земли можно рассматривать как “вечное” падение. От фактического падения его “удерживает” только то, что вектор скорости всегда направлен перпендикулярно радиусу окружности вращения. Действительно, именно из-за такого “вечного” падения космонавты испытывают чувство невесомости. Дело в том, что космонавты и их космический корабль “вечно” падают по касательной к орбите вращения вокруг Земли, но при этом нисколько не приближаются к Земле.

В практических целях часто важнее знать период обращения искусственного спутника, а не его скорость. Это нужно, например, в ситуации, когда требуется определить момент выхода на связь с космическим кораблем.

Вычисляем период обращения спутника

Периодом обращения спутника называется время, которое необходимо ему, чтобы совершить полный цикл вращательного движения по орбите. Если нам известна орбитальная скорость движения ​\( v \)​ спутника по окружности с радиусом ​\( r \)​ (см. предыдущий раздел), то можно легко и просто вычислить период обращения ​\( T \)​. За период обращения спутник преодолевает расстояние, равное длине окружности ​\( 2\pi r \)​. Это значит, что орбитальная скорость ​\( v \)​ спутника равна \( 2\pi r/T \). Приравнивая это соотношение и полученное ранее выражение для орбитальной скорости

где ​\( m \)​ — масса Земли, получим:

Отсюда легко получить следующее выражение для периода обращения спутника:

А на какой высоте должен находиться спутник, чтобы вращаться с периодом обращения Земли вокруг своей оси, равным 24 часам или 86400 с? Это вовсе не праздный вопрос. Такие спутники действительно существуют и используются для обеспечения непрерывной связи в данном регионе. Действительно, ведь, обращаясь вокруг Земли с тем же периодом, что и Земля, спутник на такой геостационарной орбите постоянно находится над одной и той же точкой поверхности Земли. Несколько таких спутников образуют систему глобального позиционирования. Итак, с помощью предыдущей формулы вычислим радиус окружности вращения спутника на стационарной орбите:

Подставляя численные значения, получим:

Отнимая от этой величины 4,23·107 м, значение радиуса Земли, равное 6,38·106 м, получим приблизительно 3,59·107 м, т.е. около 35900 км. Именно на таком расстоянии от Земли вращаются спутники глобальной системы позиционирования.

На практике спутники на геостационарной орбите все же теряют скорость из- за взаимодействия с магнитным полем Земли (подробнее о магнитном поле рассказывается в следующих главах). Поэтому спутники оборудованы небольшими двигателями для корректировки их положения на геостационарной орбите.

Вращаемся вдоль вертикальной плоскости

Наверняка вам приходилось наблюдать, как отважные мотоциклисты, велосипедисты или скейтбордисты вращаются внутри круглого трека, расположенного в вертикальной плоскости. Почему сила тяжести не опрокидывает их в самой верхней точке, где они находятся вверх ногами? Как быстро им нужно двигаться, чтобы сила гравитации не превышала центростремительной силы?

Рассмотрим эту ситуацию подробнее с помощью схемы на рис. 7.4. Для простоты предположим, что вместо отважных спортсменов маленький мячик совершает движение по окружности, расположенной в вертикальной плоскости. Итак, предыдущий вопрос формулируется следующим образом: “Какой минимальной скоростью должен обладать мячик, чтобы совершить полный цикл движения по вертикально расположенной окружности?”. Какому основному условию должно отвечать движение мячика, чтобы он совершил полный цикл движения по такой окружности и не упал в самой верхней точке?

Для прохождения самой верхней точки без падения мячик должен обладать минимальной скоростью, достаточной для создания такой центростремительной силы, которая была бы не меньше силы гравитации.

При таких условиях нормальная сила со стороны трека будет равна нулю, а единственной силой, которая будет удерживать объект на окружности, является сила гравитации. Поскольку центростремительная сила равна:

а сила гравитации равна:

то, приравнивая их, получим:

Отсюда получим выражение для минимально необходимой скорости для безопасного движения по окружности, расположенной в вертикальной плоскости:

Обратите внимание, что на величину минимально необходимой скорости для безопасного движения объекта по окружности, расположенной в вертикальной плоскости, не влияет масса объекта, будь-то мячик, мотоцикл или гоночный автомобиль.

Любой объект, движущийся с меньшей скоростью, в самой верхней точке трека неизбежно отклонится от траектории движения по окружности и упадет. Давайте вычислим величину минимально необходимой скорости для безопасного движения по окружности с радиусом 20 м. Подставляя численные значения в предыдущую формулу, получим:

Итак, для безопасного движения по окружности с радиусом 20 м объект (мячик, мотоцикл или гоночный автомобиль) должен иметь скорость не менее 14 м/с, т.е. около 50 км/ч.

Учтите, что для безопасного движения по окружности такую минимальную скорость объект должен иметь в самой верхней точке! Для того чтобы развить такую скорость в верхней точке, объекту в нижней точке нужно иметь гораздо большую скорость. Действительно, ведь чтобы добраться до верхней точки объекту придется какое-то время преодолевать силу гравитации с неизбежной потерей скорости.

Возникает вопрос: какую минимальную скорость в нижней точке должен иметь объект для безопасного движения по такой окружности? Подробный ответ на этот вопрос будет дан в части III этой книги, в которой рассматриваются такие понятия, как “кинетическая энергия”, “потенциальная энергия” и “преобразование энергии из одной формы в другую”.

Глава 7. Движемся по орбитам

Оценка

Угловая скорость

Положение материальной точки на окружности определяется радиусом-вектором $ \overrightarrow {r}$, проведенным из центра окружности. Модуль радиуса-вектора равен радиусу окружности R (рис. 1).

Рисунок 1. Радиус-вектор, перемещение, путь и угол поворота при движении точки по окружности

При этом движение тела по окружности можно однозначно описать с помощью таких кинематических характеристик, как угол поворота, угловая скорость и угловое ускорение.

За время ∆t тело, двигаясь из точки А в точку В, совершает перемещение $\triangle r$, равное хорде АВ, и проходит путь, равный длине дуги l. Радиус-вектор поворачивается на угол ∆$ \varphi $.

Угол поворота можно характеризовать вектором углового перемещения $d\overrightarrow{{\mathbf \varphi }}$, модуль которого равен углу поворота ∆$ \varphi $, а направление совпадает с осью вращения, причем так, что направление поворота отвечает правилу правого винта по отношению к направлению вектора $d\overrightarrow{{\mathbf \varphi }}$.

Вектор $d\overrightarrow{{\mathbf \varphi }}$ называется аксиальным вектором (или псевдо-вектором), тогда как вектор перемещения $\triangle \overrightarrow{r}$ является полярным вектором (к ним также относятся векторы скорости и ускорения). Они отличаются тем, что полярный вектор кроме длины и направления имеет точку приложения (полюс), а аксиальный вектор имеет только длину и направление (ось — по латыни axis), но не имеет точки приложения. Векторы такого типа часто применяются в физике. К ним, например, относятся все вектора, являющиеся векторным произведением двух полярных векторов.

Скалярная физическая величина, численно равная отношению угла поворота радиуса-вектора к промежутку времени, за который этот поворот произошел, называется средней угловой скоростью: $\left\langle \omega \right\rangle =\frac{\triangle \varphi }{\triangle t}$. В СИ единицей угловой скорости является радиан в секунду $( \frac {рад} {c})$.

Определение

Угловой скоростью вращения называется вектор, численно равный первой производной угла поворота тела по времени и направленный вдоль оси вращения по правилу правого винта:

\[\overrightarrow{{\mathbf \omega }}\left(t\right)={\mathop{lim}_{\triangle t\to 0} \frac{\triangle {\mathbf \varphi }}{\triangle t}=\frac{d\overrightarrow{{\mathbf \varphi }}}{dt}\ }\]

При равномерном движении по окружности угловая скорость и модуль линейной скорости — величины постоянные: ${\mathbf \omega }=const$; $v=const$.2R$

При неравномерном движении по окружности вектор угловой скорости является векторной функцией от времени $\overrightarrow{\omega }\left(t\right)={\overrightarrow{\omega }}_0+\overrightarrow{\varepsilon }\left(t\right)t$, где ${\overrightarrow{{\mathbf \omega }}}_0$ — начальная угловая скорость, $\overrightarrow{{\mathbf \varepsilon }}\left(t\right)$ — угловое ускорение. В случае равнопеременного движения, $\left|\overrightarrow{{\mathbf \varepsilon }}\left(t\right)\right|=\varepsilon =const$, и $\left|\overrightarrow{{\mathbf \omega }}\left(t\right)\right|=\omega \left(t\right)={\omega }_0+\varepsilon t$.

Задача 1

Опишите движение вращающегося твердого тела в случаях, когда угловая скорость изменяется согласно графикам 1 и 2, изображенным на рис.2.

Рисунок 2.

Решение

Вращение бывает в двух направлениях — по часовой стрелке и против. С направлением вращения связан псевдовектор угла поворота и угловой скорости. Пусть положительным будем считать направление вращения по часовой стрелке.

Для движения 1 угловая скорость возрастает, но угловое ускорение $\varepsilon $=d$\omega $/dt (производная) уменьшается, оставаясь положительным. Следовательно, это движение является ускоренным по часовой стрелке с уменьшающимся по величине ускорением.

Для движения 2 угловая скорость уменьшается, затем достигает в точке пересечения с осью абсцисс нуля, а далее становится отрицательной и возрастает по модулю. Угловое ускорение отрицательно и уменьшается по модулю. Таким образом, сначала точка двигалась по часовой стрелке замедленно с уменьшающимся по модулю угловым ускорением, остановилась и стала вращаться ускоренно с уменьшающимся по модулю ускорением.

Задача 2

Найти радиус R вращающегося колеса, если известно, что линейная скорость $v_1$ точки, лежащей на ободе, в 2,5 раза больше линейной скорости $v_2$ точки, лежащей на расстоянии $r = 5 см$ ближе к оси колеса.

Решение

Рисунок 3.

Дано:

$$R_2 = R_1 — 5$$ $$v_1 = 2,5v_2$$ $$R_1 = ?$$

Точки движутся по концентрическим окружностям, вектора их угловых скоростей равны, $\left|{\overrightarrow{\omega }}_1\right|=\left|{\overrightarrow{\omega }}_2\right|=\omega $ , следовательно, можно записать в скалярной форме:

\[v_1=\omega R_1; v_2=\omega R_2;\frac{v_1}{v_2}=\frac{\omega R_1}{\omega R_2}=\frac{R_1}{R_1-5}=2,5;;\ R_1=\frac{5\times 2,5}{1.5}=8,3\ см\ \ \]

Ответ: радиус колеса R = 8,3 см

Урок 5. поступательное движение. вращательное движение твердого тела — Физика — 10 класс

Физика, 10 класс

Урок 05. Поступательное движение. Вращательное движение твёрдого тела

Перечень вопросов, рассматриваемых на уроке:

  1. Поступательное и вращательное движение абсолютно твердого тела.
  2. Характеристики вращательного движения абсолютно твердого тела.

Глоссарий по теме

1. Абсолютно твердое тело – это тело, расстояние между любыми двумя точками которого остается постоянным при его движении.

2. Поступательным называется такое движение абсолютно твердого тела, при котором любой отрезок, соединяющий любые две точки тела, остается параллельным самому себе. Одинаковыми остаются при поступательном движении перемещение, траектория, путь, скорость, ускорение.

3. Вращением твердого тела вокруг неподвижной оси называется такое движение, при котором все точки тела описывают окружности, центры которых находятся на одной прямой перпендикулярной плоскостям этих окружностей. Сама эта прямая есть ось вращения.

4. Угол поворота – угол, на который поворачивается радиус-вектор, соединяющий центр окружности с точкой вращающегося тела.

5. Угловая скорость — отношение угла поворота φ к промежутку времени, в течение которого совершен этот поворот при равномерном движении.

6. Линейная скорость – отношение длины дуги окружности пройденной точкой тела к промежутку времени, в течение которого этот поворот совершен.

7. Период — промежуток времени, за который тело делает один полный оборот.

8. Частота обращения тела – число оборотов за единицу времени

Основная и дополнительная литература по теме урока:

Мякишев Г.Я., Буховцев Б.Б., Сотский Н.Н.. Физика.10 класс. Учебник для общеобразовательных организаций М.: Просвещение, 2016. – С. 57-61

Рымкевич А.П. Сборник задач по физике. 10-11 класс.-М.:Дрофа,2009.-С.20-22

Открытые электронные ресурсы:

http://kvant.mccme.ru/1986/11/kinematika_vrashchatelnogo_dvi.htm

Теоретический материал для самостоятельного изучения

1. Вы знаете, что в физике для упрощения исследования реальных ситуаций часто используются модели. Одной из механических моделей, используемых при описании движения и взаимодействия тел, является абсолютно твёрдое тело- тело, расстояние между любыми двумя точками которого остаётся постоянным при его движении.

2. Поступательным называется такое движение абсолютно твёрдого тела, при котором любой отрезок, соединяющий любые две точки тела, остаётся параллельным самому себе. Примером поступательного движения может служить свободное падение тел, движение лифта, поезда на прямолинейном участке дороги. При поступательном движении все точки тела описывают одинаковые траектории, совершают одинаковые перемещения, проходят одинаковые пути, в каждый момент времени имеют равные скорости и ускорения.

Для описания поступательного движения абсолютно твёрдого тела достаточно написать уравнение движения одной из его точек.

3. Вращательным движением абсолютно твёрдого тела вокруг неподвижной оси называется такое его движение, при котором все точки тела описывают окружности, центры которых находятся на одной прямой, называемой осью вращения. При этом плоскости, которым принадлежат эти окружности, перпендикулярны оси вращения.

Вращательное движение позволяет осуществить непрерывный процесс работы с использованием больших скоростей. Вращающиеся механизмы более компактны и более экономичны, так как потери энергии на преодоление сил трения качения меньше, чем на преодоление сил трения скольжения. Поэтому в современной технике вращательное движение рабочих частей машин всё более вытесняет возвратно-поступательное. Например, вместо ножовочной пилы в технике используют вращающуюся дисковую пилу, поршневые насосы в большинстве случаев вытесняются центробежными.

4. Угловой скоростью тела при равномерном вращении называется величина, равная отношению угла поворота тела ∆φ к промежутку времени ∆t, за которое этот поворот произошёл.

Будем обозначать угловую скорость греческой буквой ω (омега). Тогда по определению запишем формулу угловой скорости;

При равномерном вращательном движении угловая скорость у всех точек вращающегося тела одинаковая. Поэтому угловая скорость, так же как и угол поворота, является характеристикой движения всего вращающегося тела, а не только отдельных его частей.

Примером вращательного движения, близкого к равномерному, может служить вращение Земли вокруг своей оси.

Угловая скорость в СИ выражается в радианах в секунду (рад/с).

Один радиан – это центральный угол, опирающийся на дугу, длина которой равна радиусу окружности.

Угловая скорость положительна, если угол между радиусом вектором, определяющим положение одной из точек твердого тела, и осью ОХ увеличивается, и отрицательным, когда он уменьшается

5.Число полных оборотов за единицу времени называют частотой обращения.

Частоту обозначают греческой буквой «ню». Единица измерения частоты является секунда в минус первой степени

Время, за которое тело совершает один полный оборот, называют периодом обращения и обозначают буквой Т.

7. Связь между линейной и угловой скоростями:

8. Связь между ускорением и угловой скоростью:

Итак, мы рассмотрели два простейших движения абсолютно твердого тела – поступательное и вращательное. В жизни мы чаще встречаем сложное движение абсолютно твердого тела, однако, в этом случае любое сложное движение можно представить как сумму двух независимых движений: поступательного и вращательного.

Примеры и разбор типового тренировочного задания

  1. Ротор мощной паровой турбины делает 100 оборотов за 2 с. Определите угловую скорость.

Дано:

N=100 об.

t = 2 c

Найти: ω.

Решение:

2. Два шкива, соединенные друг с другом ремнем, вращаются вокруг неподвижных осей (см.рис). Больший шкив радиусом 20см делает 50 оборотов за 10 секунд, а частота вращения меньшего шкива 2400 оборотов в минуту. Чему равен радиус меньшего шкива? Шкивы вращаются без проскальзывания.

Дано:

Найти —

Решение:

Из условия задачи ученик видит что, шкивы соединены ремнем, следовательно, линейные скорости их равны:

но частота вращения разная.

Сокращает на 2π обе части.

Отсюда имеем:

и так, как в условии известно , то можем записать:

Отсюда находим радиус второго шкива:

Вторая неизвестная величина

Запишем формулу периода обращения для большего шкива:

так как по условию задачи нам известно число оборотов за 10 секунд.

Подставим в формулу (1) и получим конечную формулу:

Движение по кругу — Математика A-Level, редакция

Эта страница описывает движение по кругу.

Угловая скорость

Представьте, что объект движется по круговой траектории.

Угловая скорость — это скорость изменения угла (который я обозначил буквой «а»). Таким образом, он измеряет, насколько быстро объект движется по кругу.

Угловая скорость обычно измеряется в радианах в секунду (рад / с -1 ), то есть на сколько радиан проходит частица за секунду.Кроме того, его можно измерить в оборотах в секунду, т.е. сколько полных кругов объект проходит за секунду.

Существует формула, соединяющая «нормальную» скорость (обычно называемую «линейной скоростью») и угловую скорость:

где v — линейная скорость, r — радиус окружности, а w — угловая скорость.

Пример

Частица движется по окружности радиусом 10 см. Угловая скорость 2 рад с -1 .Найдите (линейную) скорость.

Нам нужен радиус в метрах, то есть 0,1 м. Используя формулу выше, получаем:

v = 0,1 × 2 = 0,2

Значит скорость 0,2 мс -1 .

Обратите внимание, что если вам дана угловая скорость в оборотах в секунду, вам нужно сначала преобразовать в радианы в секунду. Для этого помните, что 1 оборот в секунду равен 2p радианам в секунду, потому что в круге 2p радиан.

Радиальное ускорение

Если тело движется по окружности, даже если оно движется с постоянной скоростью, оно ускоряется.Это потому, что он меняет направление (не движется по прямой).

Направление этого ускорения — к центру круга, а его величина определяется выражением:

где v — скорость, а r — радиус окружности.

Используя нашу формулу выше, это также можно записать как:

Какие из них вы будете использовать, будет зависеть от того, имеете ли вы дело со скоростью или угловой скоростью.

Ускорение происходит из-за действующей силы:

Представьте, что вы едете в машине, которая быстро свернет за поворот налево.Вы почувствуете, как сила тянет вас в сторону (левую сторону). Это сила, вызывающая ускорение. Сила действует по направлению к центру круга.

Конический маятник

Конический маятник выглядит примерно так:

P — частица. AP — это строка. P движется по синему кругу с угловой скоростью w.

Пример

Предположим, что у нас есть конический маятник, как указано выше, где частица имеет массу 2 кг, радиус круга, по которому движется частица, равен 0.5 м, а угол А составляет 45 градусов. Найдите угловую скорость P.

Вес 2 г (W = мг), где g — ускорение свободного падения.

Вертикальное разрешение: Tcos45 = 2g
Следовательно (√2T) / 2 = 2g, поэтому T = 2√2 g (1)

Теперь используйте 2-й закон Ньютона, чтобы найти уравнение движения в радиальном направлении:
(«F = m r w 2 «)
Tsin45 = 2 × 5 × w 2

Используйте (1), чтобы исключить T:
2√2 g × (√2) / 2 = 10w 2
g / 5 = w 2
Таким образом, w = √ (g / 5)

Принимая g = 9.8, мы находим, что угловая скорость составляет 1,4 рад с -1

Движение на береговой поверхности

Теперь рассмотрим движение частицы по «наклонной поверхности». Под этим я имею в виду, например, кольцевую гоночную трассу, которая наклонена вверх от центра, чтобы автомобили / мотоциклы не отставали от трассы на высоких скоростях.

Так вот, если машина едет очень быстро, она будет скользить по склону, двигаясь по кругу. Если он будет двигаться медленно, он поскользнется.

Если автомобиль не имеет тенденции к скольжению, силы и ускорение, действующие на кузов, будут такими, как на этой диаграмме (сила трения отсутствует):

Однако, если бы машина двигалась быстрее, она бы соскользнула по склону при движении по трассе. Поэтому сила трения будет действовать, пытаясь предотвратить это:

Honors Rotational Kinematics

Радианы и градусы

Обычно люди обсуждают вращательное движение в градусах, где один полный оборот по окружности равен 360 °.Когда речь идет о вращательном движении с точки зрения физики, измерение вращательного движения в единицах, известных как радианы (рад), является гораздо более эффективным. Радиан измеряет расстояние вокруг дуги, равное длине радиуса дуги.

До этого момента вы описывали расстояния и смещения в терминах Δx и Δy. Обсуждая угловые смещения, вы должны перейти к описанию поступательного смещения вокруг дуги в терминах переменной s, продолжая использовать символ θ (тета) для обозначения углов и углового смещения.

Расстояние полностью по круговой траектории (360 °), известное как окружность, C, можно найти с помощью ∆s = C = 2𝜋r = 2𝜋 радиан. Следовательно, вы можете использовать это как коэффициент преобразования для перехода между градусами и радианами.

Вопрос: Преобразовать 90 ° в радианы.

Ответ:

Вопрос: Преобразовать 6 радианов в градусы.

Ответ:

Углы также измеряются в оборотах (полные обходы по кругу).Полный единичный оборот равен 360 °, поэтому вы можете записать коэффициенты пересчета для углов поворота и смещений как 360 ° = 2 радиан = 1 оборот.

Вопрос: Преобразовать 1,5 оборота в радианы и градусы.

Ответ:

Скорость и ускорение

Вращательная кинематика очень похожа на поступательную кинематику, все, что вам нужно сделать, это изучить вращательные версии кинематических переменных и уравнений.Когда вы изучали кинематику перевода, смещение обсуждалось в терминах Δx. При вращательной кинематике вместо этого вы будете использовать угловую координату θ. Когда в трансляционном мире была введена средняя скорость, вы использовали формулу:

Изучая вращательное движение, вы будете говорить об угловой скорости ω (омега), выраженной в радианах в секунду (рад / с). Поскольку угловая (вращательная) скорость является вектором, определите положительное направление вращения как вращение против часовой стрелки по круговой траектории, а отрицательное направление как по часовой стрелке вокруг траектории.

ПРИМЕЧАНИЕ: Формально направление угловых векторов определяется правилом правой руки. Оберните пальцы правой руки в направлении вращения, скорости или ускорения, а большой палец будет указывать в направлении вектора.

Вопрос: Рекорд вращается на фонографе со скоростью 33 об / мин (оборотов в минуту) по часовой стрелке.Найдите угловую скорость записи.

Ответ:

Обратите внимание, что вектор угловой скорости отрицательный, потому что запись вращается по часовой стрелке.

Вопрос: Найдите величину угловой скорости Земли в радианах в секунду.

Ответ: Осознавая, что Земля совершает один полный оборот каждые 24 часа, мы можем оценить величину угловой скорости Земли как:

Аналогичным образом, когда вы узнали о поступательном ускорении, вы нашли ускорение как скорость изменения поступательной скорости объекта:

Угловое ускорение α (альфа), выраженное в радианах в секунду. 2 — это скорость изменения угловой скорости объекта.Поскольку угловое ускорение также является вектором, вы можете определить его направление как положительное для увеличения угловой скорости в направлении против часовой стрелки и отрицательное для увеличения угловой скорости в направлении по часовой стрелке.

Вопрос: Лягушка едет на одноколесном велосипеде. Если колесо моноцикла начинается в состоянии покоя и равномерно ускоряется против часовой стрелки до угловой скорости 15 об / мин за 6 секунд, найдите угловое ускорение колеса моноцикла.

Ответ: Сначала преобразуйте 15 об / мин в рад / с.

Далее используйте определение углового ускорения.

Снова обратите внимание на положительное угловое ускорение, поскольку колесо велосипеда ускоряется против часовой стрелки.

Вращательные переменные

Объединяя эти определения, вы наблюдаете очень сильную параллель между поступательными кинематическими величинами и вращательными кинематическими величинами.

Переменная Трансляционный Угловой
Рабочий объем

Δs

Δθ

Скорость

v

ω

Разгон

а

α

Время

т

т

Также довольно просто выполнить перевод между поступательными и угловыми переменными, когда вы знаете радиус (r) интересующей точки на вращающемся объекте.

Вопрос: Рыцарь взмахивает булавой радиусом 1 м за два полных оборота. Что такое поступательное смещение булавы?

Ответ:

Вопрос: Проигрыватель компакт-дисков предназначен для изменения скорости вращения диска, так что точка, считываемая лазером, перемещается с линейной скоростью, равной единице.25 м / с. Какова скорость вращения компакт-диска в об / с, когда лазер считывает информацию с внутренней части диска на радиусе 0,03 м?

Ответ:

Вопрос: Какова скорость вращения компакт-диска в предыдущей задаче, когда лазер считывает крайнюю часть диска (радиус = 0,06 м)?

Ответ:

Вращательные кинематические уравнения

Параллели между поступательным и вращательным движением идут еще дальше.Вы разработали набор кинематических уравнений поступательного движения, которые позволили вам исследовать взаимосвязь между смещением, скоростью и ускорением. Вы можете разработать соответствующий набор соотношений для углового смещения, угловой скорости и углового ускорения. Уравнения имеют ту же форму, что и трансляционные уравнения, все, что вам нужно сделать, это заменить трансляционные переменные на вращательные переменные, как показано в следующей таблице.

Уравнения вращательной кинематики можно использовать так же, как вы использовали уравнения поступательной кинематики для решения задач.Когда вы знаете три кинематические переменные, вы всегда можете использовать уравнения для решения двух других.

Вопрос: Плотник вырезает кусок дерева с помощью мощной циркулярной пилы. Пильный диск ускоряется из состояния покоя с угловым ускорением от 14 рад / с 2 до максимальной скорости 15 000 об / мин. Какова максимальная скорость пилы в радианах в секунду?

Ответ:

Вопрос: Сколько времени нужно пиле для достижения максимальной скорости?

Ответ: Для решения этой задачи можно использовать уравнения кинематики вращения:

Вопрос: Сколько полных оборотов делает пила при разгоне до максимальной скорости?

Ответ:

Вопрос: Предохранительный механизм остановит пильный диск в 0.3 секунды, если рука плотника оторвется от органов управления пилой. Какое угловое ускорение для этого требуется? Сколько полных оборотов пильное полотно сделает за это время?

Ответ: Начните с воссоздания таблицы кинематики вращения.

Сначала найдите угловое ускорение.

Затем найдите угловое смещение.

Наконец, преобразуйте угловое смещение в число оборотов.

Угловая скорость

Привет, Бен.

Линейная скорость — это расстояние, пройденное по прямой за единицу времени. Угловая скорость — это угол, пройденный за единицу времени.

Самый простой пример — это равномерное круговое движение. Например, камешек, застрявший в покрышке велосипеда, движется равномерно по кругу.

Допустим, внешний диаметр велосипедной шины составляет 70 см, и вы двигаетесь со скоростью 40 км / ч. Помните, что (игнорируя занос) шина всегда сохраняет сцепление с дорогой, поэтому расстояние, которое вы проедете, и скорость, которую вы путешествуете, совпадают со скоростью шины. Если преобразовать эту скорость в метры в секунду, мы получим:

.

Это скорость велосипеда, значит, это линейная скорость велосипеда. Поскольку байк и шина находятся в постоянном контакте, это также линейная скорость гальки в шине.

Угловая скорость связана с углами, как подсказывает название. Выражается в том, на какой угол поворачивается за определенный промежуток времени.

Углы можно измерить разными способами: вращение, градусы и радианы. Например, жесткий диск на 10000 об / мин относится к его угловой скорости: 10000 оборотов в минуту. Итак, насколько быстро вращается наш камешек?

Этот камешек, как мы знаем, движется со скоростью 11,11 м / с линейно, но движется по окружности шины.Если внешний диаметр шины d составляет 70 см, то длина окружности составляет πd , что составляет около 2,199 м.

Если мы разделим линейную скорость на длину окружности шины, мы узнаем, сколько оборотов в секунду совершает эта шина (и, следовательно, галька) — под углом:

, где R = обороты.

Это угловая скорость: сколько она оборотов в единицу времени.Если вы хотите преобразовать это в градусы, просто умножьте на количество градусов на один оборот (360), если вы хотите вместо этого использовать радианы, умножьте на 2π радиан на один оборот.

А теперь попробуем другое направление. Допустим, вы знаете угловую скорость и пытаетесь вычислить соответствующую линейную скорость. Подсказка: это довольно быстро!

Возьмем для примера вращение Земли. Вы знаете, какова его угловая скорость: сколько он поворачивается в единицу времени.Очевидно, это одна ротация в день!

Давайте спросим себя, какова линейная скорость шлюза на Панамском канале. Это достаточно близко к экватору, чтобы мы могли использовать диаметр Земли на экваторе (12756 км) в качестве ориентира. Какова окружность Земли в этой точке?

Если вы вычислите длину окружности, вы можете умножить ее на угловую скорость, и вы получите линейную скорость.

Вот кое-что действительно интересное. : Поскольку угловая скорость зависит от длины окружности (и, как следствие, радиуса), вы можете иметь что-то, что движется с одинаковой угловой скоростью, но с разными линейными скоростями.

Давайте подумаем о (передней) звездочке, установленной на вашем велосипеде. Если вы перейдете с маленькой звездочки на большую и будете двигать ногами с одинаковым числом оборотов в минуту (с одинаковой угловой скоростью), то вы будете двигаться быстрее. Это потому, что линейная скорость цепи — это то, что движет скоростью велосипеда. Таким образом, когда вы меняли шестерни, вы переходили от звездочки с малым радиусом (и, следовательно, с малой окружностью) к звездочке с большим радиусом (окружностью).Когда вы умножаете эти два значения на одинаковую угловую скорость, вы получаете большую линейную скорость с большой звездочкой!

Значит, большая звездочка с двойным радиусом маленькой звездочки должна заставить вас преодолеть вдвое большее расстояние за такое же количество оборотов ног.

Суммируем:

  • Разделите линейную скорость на длину окружности, чтобы получить угловую скорость (в оборотах за единицу времени, которые затем можно преобразовать в любые другие единицы, которые вас интересуют).
  • Умножьте угловую скорость (в оборотах в единицу времени — преобразуйте сначала, если нужно) на длину окружности, чтобы получить линейную скорость.

Надеюсь, это проясняет вам это!
Стивен Ла Рок>

линейных и угловых скоростей, площади секторов и длины дуг — она ​​любит математику

Этот раздел охватывает:

Примерно в то время, когда вы узнаете о радианах и , вам, возможно, придется работать с линейными и угловыми скоростями , а также областями секторов и длинами дуг .

Примечание : Для целей этого раздела мы будем говорить о линейной скорости и угловой скорости ( скалярных величин), в отличие от линейной скорости и угловой скорости ( векторных величин). .

Мы обсудили радиан и то, как они связаны с градусами центральных углов здесь, в разделе углов и единичной окружности . Прежде чем говорить о линейных и угловых скоростях, давайте рассмотрим, как радианы связаны с длиной окружности, а также с оборотами окружности.

Обратите внимание, что \ (\ theta \) — это центральный угол поворота в радиан, , \ (r \) — радиус окружности, \ (s \) — длина дуги, или пересеченная дуга (часть окружность) круга, а оборотов (или оборотов ) — это когда объект полностью прошел по кругу (или круг вернется в исходное положение). Все эти единицы могут относиться к окружности окружности: \ (2 \ pi r \), где \ (r \) — радиус.(Помните, что в единичном круге длина окружности равна просто \ (2 \ pi \), поскольку \ (r = 1 \)).

Обратите внимание, что слова «радиан» и «радиус» связаны между собой, поскольку в одном обороте \ (2 \ pi r \) (радиан), а \ (2 \ pi r \) (радиус) — это измерение длины окружности. .

Одна из наиболее важных концепций состоит в том, что длина пересеченной дуги равна радиусу , в раз превышающему радиан измерения центрального угла этой дуги . Чтобы увидеть это, установите пропорцию, сравнивая эту дугу со всей окружностью:

\ (\ displaystyle \ require {cancel} \ frac {{\ text {Длина дуги}}} {{\ text {Circumference}}} = \ frac {{\ text {Arc} \! \! ‘\! \ ! \ text {s Angle}}} {{\ text {Измерение общего угла в круге}}}: \, \, \, \, \ frac {s} {{\ cancel {{2 \ pi}} r}} = \ frac {\ theta} {{\ cancel {{2 \ pi}}}} \)

Отсюда видно, что \ (s = r \ theta \).

Я даю вам несколько основных формул, но, честно говоря, когда я решаю большинство этих задач, я просто использую множители единиц (размерный анализ), чтобы найти единицы, необходимые для решения задачи !!

Формулы линейной и угловой скорости:

Мы будем использовать эти формулы в некоторых задачах линейной / угловой скорости ниже.

Помните, что вы обычно используете радиус с линейной скоростью и радиан с угловой скоростью дюймов \ (2 \ pi r \).

Линейная скорость — это скорость, с которой точка на внешней стороне объекта движется по круговой траектории вокруг центра этого объекта. Это могут быть любые обычные единицы скорости, такие как мили в час, метры в секунду и так далее.

Мы помним, что \ (\ text {Distance} = \ text {Rate} \ times \ text {Time} \) или \ (\ displaystyle \ text {Rate (Speed)} = \ frac {{\ text {Distance}} }} {{\ text {Time}}} \). Сначала мы поговорим о том, как быстро изменяется объект по окружности круга.

Представьте себе автомобиль, который движется по кругу по рельсовому пути с длиной дуги (фактическая длина извилистой части — часть окружности ) \ (s \). Формула для скорости по кругу или линейной скорости : \ (\ displaystyle v = \ frac {s} {t} \), где \ (s \) — длина дуги, а \ (t \) — время.

Вот тип проблемы, которая может у вас возникнуть. Обратите внимание, что мы должны использовать множителей единиц (размерный анализ), когда единицы не совпадают.

Задача линейной скорости Решение
Автомобиль движется с постоянной скоростью по круговой трассе с окружностью 4 миль.

Если автомобиль проходит 8 кругов за 10 минут, какова линейная скорость автомобиля в милях в час?

Давайте сначала посчитаем расстояние, которое проехала машина. Если окружность окружности составляет 4 миль, а автомобиль проехал 8 кругов, то автомобиль пройдет в общей сложности 32 миль:

\ (\ require {cancel} \ displaystyle 8 \ cancel {{\ text {laps}}} \ cdot \ frac {{4 \, \, \ text {miles}}} {{\ text {1} \ cancel { {\ text {lap}}}}} = 32 \, \, \ text {miles} \)

Теперь давайте получим линейную скорость по формуле \ (\ displaystyle v = \ frac {s} {t} \).Мы знаем \ (s = 32 \) миль и \ (t = 10 \) минут. Но мы должны добавить несколько множителей единиц, так как у нас есть время в минут , и мы хотим получить миль в час :

\ (\ displaystyle v = \ frac {s} {t} = \ frac {{32 \, \, \ text {miles}}} {{10 \, \, \ cancel {{\ text {minutes}}}} }} \ cdot \ frac {{60 \, \, \ cancel {{\ text {minutes}}}}} {{1 \, \, \ text {hour}}} = 192 \, \, \ text {миль / час} \)

Линейная скорость автомобиля 192 миль в час .Видите, как мы могли бы это выяснить без формулы, а просто с использованием единичных множителей?

Колесо диаметром 10 дюймов вращается с постоянной скоростью 2 оборотов в секунду.

Найдите линейную скорость колеса в милях в час ( 1 миль составляет приблизительно 5 280 футов).

Мы хотим закончить с миль в час , поэтому давайте поместим это в конец нашего уравнения множителя ; мы знаем, что нам нужны мили наверху и часы где-то внизу):

\ (\ displaystyle… \, \, \ frac {?} {\ Text {?}} \ Cdot \ frac {?} {?} \ Cdot \ frac {?} {{\ Text {?} \, \, \ text {часы}}} \ cdot \ frac {{? \, \, \, \ text {miles}}} {?} \, \,… =? \, \, \ text {миль в час} \)

Давайте создадим соотношения всего остального, что мы знаем, и убедимся, что можем вычеркнуть все, что нам не нужно.Нам нужно знать, что один оборот круга эквивалентен \ (\ pi d \) дюйму. Вы можете начать с более короткого уравнения и медленно добавлять множители:

\ (\ displaystyle… \, \, \ frac {{2 \, \, \ cancel {{\ text {Revolutions}}}}} {{\ text {1} \, \, \ text {second} \, }} \ cdot \ frac {{\ pi d}} {{1 \, \, \ cancel {{\ text {Revolution}}}}} \ cdot \ frac {?} {{\ text {?} \, \ , \ text {часы}}} \ cdot \ frac {{? \, \, \, \ text {miles}}} {?} \, \,… =? \, \, \ text {миль в час} \ )

\ (\ Displaystyle \ frac {{2 \, \, \ cancel {{\ text {Revolutions}}}}} {{\ text {1} \, \, \ cancel {{\ text {second}}} \ ,}} \ cdot \ frac {{\ pi \ left ({10} \ right) \, \, \ cancel {{\ text {дюймы}}}}} {{1 \, \, \ cancel {{\ text {Revolution}}}}} \ cdot \ frac {{60 \, \, \ cancel {{\ text {min}}}}}} {{\ text {1} \, \, \, \ text {hour}} } \ cdot \ frac {{60 \, \, \ cancel {{\ text {секунды}}}}} {{1 \, \, \ cancel {{\ min}}}} \ cdot \ frac {{1 \ , \, \, \ text {mile}}} {{5280 \, \, \ cancel {{\ text {ft}}}}} \ cdot \ frac {{1 \, \, \ cancel {{\ text { ft}}}}} {{12 \, \, \ cancel {{\ text {дюймы}}}}} \)

\ (\ displaystyle \, = \ frac {{2 \ cdot \ pi \ left ({10} \ right) \ cdot 60 \ cdot 60}} {{5280 \ cdot 12}} \, \, \ text {миль в час} \ примерно 3.57 \, \, \ text {миль в час} \)

Линейная скорость колеса 3,57 миль в час .

Угловая скорость — это скорость вращения объекта, описываемая в таких единицах, как обороты в минуту, градусы в секунду, радианы в час и т. Д.

Угловая скорость связана с тем, насколько быстро изменяется центральный угол окружности, а не длина окружности.

Снова представьте себе машину, которая движется по кругу по рельсовому пути с центральным углом \ (\ theta \).Формула для скорости по окружности через этот угол, или угловая скорость , равна \ (\ displaystyle \ omega = \ frac {\ theta} {t} \), где \ (\ theta \) в радианах. , а \ (t \) — время.

Вот тип проблемы, которая может у вас возникнуть. Обратите внимание, что мы должны снова использовать множителей единиц (размерный анализ), когда единицы не совпадают.

Задача угловой скорости Решение
Хизер мигает фонариком и вращается по кругу с постоянной скоростью.

Если фонарик Хизер совершает один оборот (оборот) каждые 15 секунд, какова угловая скорость света, исходящего от фонарика, в радианах в минуту?

Поскольку задача включает в себя информацию о времени, которое требуется на один оборот (оборот), нам нужно использовать тот факт, что на один оборот приходится \ (2 \ pi \) радиан.

Теперь давайте воспользуемся нашим уравнением угловой скорости \ (\ displaystyle \ omega = \ frac {{\ theta \ text {(радианы)}}} {t} \) с некоторыми множителями единиц .

Начните с того, что мы знаем, и закончите тем, что нам нужно знать (нам нужны радианы в ответе):

\ (\ require {cancel} \ displaystyle \ omega = \ frac {\ theta} {t} = \ frac {{\ cancel {{1 \ text {вращение}}}}} {{15 \, \, \ cancel {{\ text {секунды}}}}} \ cdot \ frac {{2 \ pi \ text {радианы}}} {{\ cancel {{1 \ text {вращение}}}}} \ cdot \ frac {{60 \, \, \ cancel {{\ text {секунды}}}}} {{1 \, \, \ text {minute}}} = \, 8 \ pi \, \, \ text {радианы / минута} \)

Угловая скорость света составляет \ (8 \ pi \) радиан в минуту. Опять же, посмотрите, как мы могли бы вычислить это без формулы, а просто используя единичные множители?

Вот пример, который показывает разницу между определением угловой и линейной скоростей. Обратите внимание, что при угловых скоростях мы игнорируем радиус , так как мы имеем дело только с поворотом на угол. Также обратите внимание, что:

\ (\ text {Linear Speed} = \ text {Radius} \ times \ text {Angular Speed} \) или \ (\ displaystyle \ text {Radius} = \ frac {{\ text {Linear Speed}}} { {\ text {Угловая скорость}}} \)

Задачи линейной и угловой скорости Решение
Джени сидит на карусели, она находится на расстоянии 6 футов от центра.

Найдите ее угловых и линейных скоростей, если карусель движется со скоростью 5 оборотов в минуту.

Обратите внимание, что:

\ (\ displaystyle \ text {radius} = \ frac {{\ text {linear speed}}} {{\ text {angular speed}}} \)

Уравнение для угловой скорости : \ (\ displaystyle \ omega = \ frac {{\ theta \ text {(в радианах)}}} {t} \), но нам уже дана скорость в оборотах в минуту.Преобразуйте эту скорость в угловую скорость, используя единичные множители (зная, что за один оборот приходится \ (2 \ pi \) радиан):

\ (\ require {cancel} \ displaystyle \ frac {{5 \ text {} \ cancel {{\ text {Revolutions}}}}} {{\ text {minute}}} \ cdot \ frac {{2 \ pi \ text {радианы}}} {{1 \ text {} \ cancel {{\ text {Revolution}}}}} = 10 \ pi \, \, \ text {радиан в минуту} \).

Обратите внимание, что мы не используем радиус, поскольку нас интересует только угловая скорость.

Чтобы получить линейную скорость , давайте снова воспользуемся единичными множителями, но теперь мы должны взглянуть на радиус (поскольку нам нужно расстояние по окружности круга с этим радиусом).Вы также можете помнить, что \ (s \) (длина дуги или часть окружности) равна \ (2 \ pi r \) на один оборот:

\ (\ displaystyle \ begin {align} \ frac {{5 \ text {} \ cancel {{\ text {Revolutions}}}}} {{\ text {minute}}} \ cdot \ frac {{2 \ pi r \ text {feet}}} {{1 \ text {} \ cancel {{\ text {Revolution}}}}} & = \ frac {{5 \ text {} \ cancel {{\ text {Revolutions}}} }} {{\ text {minute}}} \ cdot \ frac {{2 \ pi \ left (6 \ right) \ text {feet}}} {{1 \ text {} \ cancel {{\ text {Revolution} }}}} \\ & = 60 \ pi \, \ text {футов в минуту} \ end {align} \)

(Мы также можем использовать формулу \ (\ text {Линейная скорость} = \ text {Радиус} \ times \ text {Угловая скорость} \), чтобы получить \ (60 \ pi \).)

Вот еще проблемы с линейными и угловыми скоростями и оборотов / оборотов :

Задачи линейной и угловой скорости Решение
Бри хочет запрыгнуть на движущуюся карусель диаметром 50 футов и двигаться со скоростью 3 оборотов в минуту.

Насколько быстро должна бежать Бри, чтобы соответствовать скорости карусели, чтобы прыгнуть (в футах в секунду)?

Поскольку мы хотим знать, с какой скоростью должна бежать Бри, мы ищем линейную скорость .Но нам дана скорость 3 оборотов в минуту, поэтому давайте просто свяжем скорости друг с другом.

Мы можем начать с того, что мы знаем, и, используя множители единиц, начать заполнять единицы, помещать единицы в числитель или знаменатель, в зависимости от того, хотим ли мы, чтобы они оставались или вычеркиваем их. Помните, что для каждого поворота или поворота линейное расстояние или длина дуги \ (s = 2 \ pi r \), где \ (\ displaystyle r = \ frac {{\ text {Diameter}}} {2} \):

\ (\ require {cancel} \ displaystyle \ begin {align} \ frac {{3 \ text {Revolutions}}} {{1 \ text {minute}}} \ cdot \ frac {?} {?} \ Cdot \ frac {{2 \ pi \ left ({25} \ right) \, \, \ text {feet}}} {{1 \ text {Revolution}}} & = \ frac {{\ text {feet}}} { {\ text {second}}} \\\ frac {{3 \, \, \ cancel {{\ text {Revolutions}}}}} {{1 \, \, \ cancel {{\ text {minute}}} }} \ cdot \ frac {{1 \, \, \ cancel {{\ text {minute}}}}} {{60 \, \, \ text {seconds}}} \ cdot \ frac {{50 \ pi \ , \ text {feet}}} {{1 \, \, \, \ cancel {{\ text {Revolution}}}}} & = \ frac {{3 \ cdot 50 \ pi \ text {}}} {{ \ text {60}}} \ frac {{\ text {футов}}} {{\ text {second}}} \\ & = 5 \ pi \, \, \ text {футов в секунду} \\ & \ приблизительно \ text {14 футов / сек} \ end {align} \)

Бри нужно бегать со скоростью 14 футов в секунду , чтобы запрыгнуть на карусель . Это быстро!

Что такое вращение в оборотах в минуту при угловой скорости , равной 50 радиан в час? У нас есть угловая скорость 50 радиан в час, но нам нужна скорость вращения в оборотах в минуту.

Мы можем начать с того, что мы знаем, и, используя множители единиц, начать заполнять единицы, помещать единицы в числитель или знаменатель, в зависимости от того, хотим ли мы, чтобы они оставались или вычеркиваем их:

\ (\ begin {align} \ frac {{50 \ text {радианы}}} {{1 \ text {hour}}} \ cdot \ frac {?} {?} \ Cdot \ frac {?} {?} & = \ frac {{\ text {Revolutions}}} {{\ text {minute}}} \\\ frac {{50 \ text {} \ cancel {{\ text {radians}}}}} {{\ cancel {{1 \ text {час}}}}} \ cdot \ frac {{\ text {1 революция}}} {{2 \ pi \, \, \, \ cancel {{\ text {радианы}}}}} \ cdot \ frac {{\ cancel {{1 \, \, \ text {hour}}}}} {{\ text {60 минут}}} & = \ frac {{50}} {{2 \ pi \ cdot 60}} \ frac {{\ text {Revolutions}}} {{\ text {minute}}} \\ & = \ frac {5} {{12 \ pi}} \, \, \, \ text {оборотов на минута} \ end {align} \)

Вращение равно \ (\ displaystyle \ frac {5} {{12 \ pi}} \) или .133 оборотов в минуту .

Каков радиус окружности с линейной скоростью 100 дюймов в час и ,1 оборотов в минуту? Давайте воспользуемся формулой, которая связывает радиус с линейной и угловой скоростями : \ (\ displaystyle \ text {radius} = \ frac {{\ text {linear speed}}} {{\ text {угловая скорость}}} = \ frac {v} {\ omega} \). Мы также должны использовать множители единиц для изменения единиц, так как у них есть часы и минуты.Мы также знаем, что один оборот равен \ (2 \ pi \) (и радианы могут быть без единиц измерения). У нас есть:

\ (\ displaystyle \ begin {align} \ text {radius} & = \ frac {v} {\ omega} = \ frac {{\ frac {{100 \, \, \ text {дюймы}}} {{1 \, \, \ text {час}}}}} {{\ frac {{. 1 \, \, \ text {Revolution}}} {{1 \, \, \ text {minute}}}}} \\ & = \ frac {{100 \, \, \ text {дюймы}}} {{1 \, \, \ cancel {{\ text {hour}}}}} \ cdot \ frac {{1 \, \, \ cancel {{\ text {minute}}}}} {{. 1 \, \, \ cancel {{\ text {Revolution}}}}} \ cdot \ frac {{1 \, \, \ cancel {{\ text {час}}}}} {{60 \, \, \ cancel {{\ text {minutes}}}}} \ cdot \ frac {{1 \, \, \ cancel {{\ text {революция}}}} } {{2 \ pi}} \\ & = \ frac {{100}} {{.1 \ cdot 60 \ cdot 2 \ pi}} \ end {align} \)

Радиус равен \ (\ displaystyle \ frac {{25}} {{3 \ pi}} \), или примерно 2,65 дюйма .

Примечание : Мы могли бы сделать это и без формулы, используя единичные множители. Начните с того, что мы знаем, и закончите с тем, что нам нужно знать (нам нужны дюймы в ответ, поэтому нам нужны дюймы сверху):

\ (\ displaystyle \ frac {{100 \, \, \ text {дюймы}}} {{1 \, \, \ cancel {{\ text {hour}}}}}} \ cdot \ frac {{\ cancel { {1 \ text {Revolution}}}}} {{2 \ pi \ text {радианы}}} \ cdot \ frac {{1 \ text {} \ cancel {{\ text {minute}}}}} {{.1 \ text {} \ cancel {{\ text {Revolution}}}}} \ cdot \ frac {{1 \, \, \ cancel {{\ text {hour}}}}} {{60 \, \, \ cancel {{\ text {minutes}}}}} = \, \ frac {{25}} {{3 \ pi}} \, \, \ text {дюймы} \)

Давайте получим площадь сектора окружности на основе радиуса и центрального угла в радианах. {2}} \ theta = \ frac {1} {2} {{\ left ({3.{2}} \ left ({\ frac {{13 \ pi}} {9}} \ right) = 510,51 \, \, \ text {ft} \)

Уравнение для площади сектора: \ (\ displaystyle s = \, r \ theta \), но мы должны преобразовать наши градусы в радианы (в 180 ° есть \ (\ pi \) радианы).

У нас:

\ (\ displaystyle 260 \, \, \ cancel {{\ text {градусов}}} \ cdot \ frac {{\ pi \, \, \ text {radians}}} {{180 \, \, \ cancel { {\ text {градусы}}}}} = \ frac {{13 \ pi}} {9} \, \ text {радианы} \).

Теперь мы можем использовать уравнение:

\ (\ displaystyle s = r \ theta = \ left ({15} \ right) \ left ({\ frac {{13 \ pi}} {9}} \ right) = 68.1 \, \, \ text {ft} \)

Разберитесь в этих проблемах и практикуйтесь, практикуйтесь, практикуйтесь!

Переходим к графикам триггерных функций — готово!

Вращательное движение жесткого тела

Дверь легче открыть, нажав на край, наиболее удаленный от петель, чем нажав посередине. Интуитивно понятно, что величина приложенной силы и расстояние от точки приложения до петли влияют на склонность двери к повороту.Эта физическая величина, крутящий момент , — это t = r × F sin θ, где F — приложенная сила, r — расстояние от точки приложения до центра вращения, а θ — угол от r до F .

Подставьте второй закон Ньютона в определение крутящего момента с θ 90 градусов (прямой угол между F и r ) и используйте соотношение между линейным ускорением и тангенциальным угловым ускорением, чтобы получить t = r F. = rma = mr 2 ( a / r ) = mr 2 α.Величина mr 2 определяется как момент инерции точечной массы относительно центра вращения.

Представьте себе два объекта одинаковой массы с различным распределением этой массы. Первый объект может быть тяжелым кольцом, поддерживаемым стойками на оси, подобной маховику. Второй объект мог иметь массу, близкую к центральной оси. Несмотря на то, что массы двух объектов равны, интуитивно понятно, что маховик будет труднее нажимать на большое количество оборотов в секунду, потому что не только количество массы, но и ее распределение влияет на легкость запуска. вращение для твердого тела.Общее определение момента инерции, также называемого инерцией вращения, для твердого тела составляет I = ∑ м i r i 2 и измеряется в единицах СИ, килограмм-метрах. 2 .

Моменты инерции для различных правильных форм показаны на рисунке 2.

Рисунок 2

Моменты инерции для различных правильных форм.

Проблемы механики часто включают как линейные, так и вращательные движения.

Пример 1: Рассмотрим рисунок 3, где груз висит на веревке, обернутой вокруг шкива. Падающая масса (м) заставляет шкив вращаться, и больше нет необходимости требовать, чтобы шкив был безмассовым. Присвойте шкиву массу ( M ) и рассматривайте его как вращающийся диск с радиусом (R) . Каково ускорение падающей массы и каково натяжение веревки?

Рисунок 3

Висящая масса вращает шкив.

Уравнение силы для падающей массы: T мг = — ma . Натяжение каната — это сила, приложенная к краю шкива, заставляющая его вращаться. Таким образом, t = I α, или TR = (1/2) MR 2 ( a / R), что сокращается до T = (1/2) млн лет назад , где угловое ускорение было заменено на a / R, потому что шнур не скользит, а линейное ускорение блока равно линейному ускорению обода диска.Объединение первого и последнего уравнения в этом примере приводит к

Решение:

Угловой момент — это вращательный момент, который сохраняется так же, как и линейный момент. Для твердого тела угловой момент (L) является произведением момента инерции и угловой скорости: L = I ω. Для точки массы угловой момент может быть выражен как произведение количества движения и радиуса ( r ): L = mvr . L измеряется в килограммах-метрах 2 в секунду или, как правило, в джоуль-секундах. Закон сохранения углового момента может быть утвержден, что угловой момент системы объектов сохраняется, если на систему не действует внешний чистый крутящий момент.

Аналогично закону Ньютона (F = Δ ( mv ) / Δ t ) существует вращательный аналог для вращательного движения: t = Δ L / Δ t , или крутящий момент — это скорость изменения углового момента.

Рассмотрим пример ребенка, который бежит по касательной к краю карусели на игровой площадке со скоростью v o и прыгает дальше, пока карусель находится в состоянии покоя. Единственными внешними силами являются сила тяжести и контактные силы, создаваемые опорными подшипниками, ни одна из которых не вызывает крутящий момент, потому что они не прикладываются, чтобы вызвать горизонтальное вращение. Рассматривайте массу ребенка как точку массы, а карусель — как диск с радиусом R и массой M .Согласно закону сохранения, полный угловой момент ребенка до взаимодействия равен полному угловому моменту ребенка и карусели после столкновения: mrv o = mrv ′ + I ω, где r — радиальное расстояние от центра карусели до места удара ребенка. Если ребенок прыгает на краю, (r = R) и угловая скорость ребенка после столкновения может быть заменена на линейную скорость, mRv o = mR ( R ω ) + (1/2) MR 2 .Если даны значения масс и начальной скорости ребенка, можно рассчитать конечную скорость ребенка и карусели.

У одиночного объекта может измениться угловая скорость из-за сохранения углового момента, если изменяется распределение массы твердого тела. Например, когда фигуристка тянет вытянутые руки, ее момент инерции уменьшается, вызывая увеличение угловой скорости. Согласно закону сохранения углового момента I o o ) = I f f ), где I o — момент инерции фигуристки с вытянутыми руками, I f — ее момент инерции с руками, расположенными близко к ее телу, ω o — ее исходная угловая скорость, а ω f — ее конечная угловая скорость.

Кинетическая энергия вращения, работа и мощность. Кинетическая энергия, работа и мощность определены в единицах вращения как K . E = (1/2) I ω 2 , W = t θ, P = t ω.

Сравнение уравнений динамики линейного и вращательного движения. Динамические соотношения приведены для сравнения уравнений линейного и вращательного движения (см. Таблицу).



Угловая скорость Земли — Вселенная сегодня

[/ caption]
Планета Земля совершает три движения: она вращается вокруг своей оси, что дает нам день и ночь; он вращается вокруг Солнца, давая нам времена года, и проходит через Млечный Путь вместе с остальной частью Солнечной системы.В каждом случае ученые пытались рассчитать не только время, необходимое для этого, но и соответствующие относительные скорости. Когда речь идет о вращении Земли вокруг своей оси, процесс, который занимает 23 часа 56 минут и 4,09 секунды, этот процесс известен как звездные сутки, а скорость, с которой она движется, известна как угловая скорость Земли. Это в равной степени относится к Земле, вращающейся вокруг оси Солнца и центра Галактики Млечный Путь.

В физике угловая скорость — это векторная величина, которая определяет угловую скорость объекта и ось, вокруг которой объект вращается.Единица измерения угловой скорости в системе СИ — радианы в секунду, хотя она может измеряться в других единицах, таких как градусы в секунду, обороты в секунду и т. Д., И обычно обозначается символом омега (ω, реже Ω). Радиан по определению — это единица измерения, которая связывает радиус дуги, длину дуги и угол, образованный дугой. Полный радиан равен 360 градусам, следовательно, мы знаем, что Земля совершает два радиана при полном вращении вокруг оси. Однако иногда ее также называют скоростью вращения, и ее величина — скорость вращения — обычно измеряется в циклах или оборотах в единицу времени (например,грамм. число оборотов в минуту). Кроме того, когда объект вращается вокруг оси, каждая точка объекта имеет одинаковую угловую скорость.

Математически средняя угловая скорость объекта может быть представлена ​​следующим уравнением: ω среднее значение = Δθ / Δt, где ω — радианы / обороты в секунду (в среднем), Δ — изменение количества, θ — скорость, а t — время. При вычислении угловой скорости Земли, когда она совершает полный оборот вокруг своей оси (солнечные сутки), это уравнение представляется как: ω avg = 2πrad / 1day (86400 секунд), что соответствует умеренным угловым значениям. скорость 7.2921159 × 10 -5 радиан / сек. В случае солнечного года, где ω ср. = 2πрад / 1 год (3,2 × 10 7 секунды), мы видим, что угловая скорость составляет 2,0 × 10 -7 рад / с.

Мы написали много статей об угловой скорости Земли для Universe Today. Вот статья об угловой скорости, а вот статья о том, почему Земля вращается.

Если вам нужна дополнительная информация об угловой скорости Земли, ознакомьтесь со следующими статьями:
Угловая скорость Земли
Вращение Земли

Мы также записали серию Astronomy Cast, посвященную планете Земля.Послушайте, Эпизод 51: Земля.

Источники:
http://en.wikipedia.org/wiki/Angular_velocity
http://hyperphysics.phy-astr.gsu.edu/hbase/rotq.html
http://hypertextbook.com/facts/2002/ JasonAtkins.shtml
http://en.wikipedia.org/wiki/Earth%27s_rotation#Rotation_period
http://www.livephysics.com/tables-of-physical-data/mechanical/angular-speed-of-earth. html

Как это:

Нравится Загрузка …

10.1 Угловое ускорение — Физика колледжа, главы 1-17

Сводка

  • Опишите равномерное круговое движение.
  • Объясните неравномерное круговое движение.
  • Вычислить угловое ускорение объекта.
  • Обратите внимание на связь между линейным и угловым ускорением.

Глава 6 Равномерное круговое движение и гравитация обсуждают только равномерное круговое движение, то есть движение по кругу с постоянной скоростью и, следовательно, с постоянной угловой скоростью. Напомним, что угловая скорость [латекс] \ boldsymbol {\ omega} [/ latex] была определена как временная скорость изменения угла [латекс] \ boldsymbol {\ theta}: [/ latex]

[латекс] \ boldsymbol {\ omega \: =} [/ latex] [латекс] \ boldsymbol {\ frac {\ Delta \ theta} {\ Delta {t}}}, [/ латекс]

, где [latex] \ boldsymbol {\ theta} [/ latex] — это угол поворота, как показано на рисунке 1.Взаимосвязь между угловой скоростью [латекс] \ boldsymbol {\ omega} [/ latex] и линейной скоростью [латекс] \ boldsymbol {v} [/ latex] также была определена в главе 6.1 «Угол поворота и угловая скорость» как

.

[латекс] \ boldsymbol {v = r \ omega} [/ латекс]

или

[латекс] \ boldsymbol {\ omega \: =} [/ latex] [латекс] \ boldsymbol {\ frac {v} {r}}, [/ latex]

, где [latex] \ boldsymbol {r} [/ latex] — это радиус кривизны, также видно на рисунке 1. Согласно соглашению о знаках, направление против часовой стрелки считается положительным направлением, а направление по часовой стрелке — отрицательным.

Рисунок 1. На этом рисунке показано равномерное круговое движение и некоторые его определенные величины.

Угловая скорость непостоянна, когда фигуристка тянет на руках, когда ребенок запускает карусель из состояния покоя или когда жесткий диск компьютера замедляется до полной остановки при выключении. Во всех этих случаях существует угловое ускорение , при котором изменяется [latex] \ boldsymbol {\ omega} [/ latex]. Чем быстрее происходит изменение, тем больше угловое ускорение. Угловое ускорение [латекс] \ boldsymbol {\ alpha} [/ latex] определяется как скорость изменения угловой скорости. 2}.2}, [/ latex] сколько времени нужно, чтобы колесо остановилось?

Стратегия для (а)

Угловое ускорение можно найти непосредственно из его определения в [latex] \ boldsymbol {\ alpha = \ frac {\ Delta \ omega} {\ Delta {t}}} [/ latex], поскольку даны окончательная угловая скорость и время . Мы видим, что [latex] \ boldsymbol {\ Delta \ omega} [/ latex] составляет 250 об / мин, а [latex] \ boldsymbol {\ Delta {t}} [/ latex] составляет 5,00 с.

Решение для (a)

Вводя известную информацию в определение углового ускорения, получаем

[латекс] \ begin {array} {lcl} \ boldsymbol {\ alpha} & \ boldsymbol {=} & \ boldsymbol {\ frac {\ Delta \ omega} {\ Delta {t}}} \\ {} & \ boldsymbol {=} & \ boldsymbol {\ frac {250 \ textbf {rpm}} {5.2} [/ latex] для углового ускорения нам нужно преобразовать [latex] \ boldsymbol {\ Delta \ omega} [/ latex] из об / мин в рад / с:

[латекс] \ begin {array} {lcl} \ boldsymbol {\ Delta \ omega} & \ boldsymbol {=} & \ boldsymbol {250 \ frac {\ textbf {rev}} {\ textbf {min}} \ cdotp \ frac {2 \ pi \ textbf {rad}} {\ textbf {rev}} \ cdotp \ frac {1 \ textbf {min}} {60 \ textbf {sec}}} \\ {} & \ boldsymbol {=} & \ boldsymbol {26.2 \ textbf {rads.}} \ end {array} [/ latex]

Вводя это количество в выражение для [latex] \ boldsymbol {\ alpha}, [/ latex], получаем

[латекс] \ begin {array} {lcl} \ boldsymbol {\ alpha} & \ boldsymbol {=} & \ boldsymbol {\ frac {\ Delta \ omega} {\ Delta {t}}} \\ {} & \ boldsymbol {=} & \ boldsymbol {\ frac {26.2} \ end {array} [/ latex]

Стратегия для (b)

В этой части мы знаем угловое ускорение и начальную угловую скорость. Мы можем найти время остановки, используя определение углового ускорения и решение для [latex] \ boldsymbol {\ Delta {t}}, [/ latex], что дает

[латекс] \ boldsymbol {\ Delta {t} \: =} [/ latex] [латекс] \ boldsymbol {\ frac {\ Delta \ omega} {\ alpha}}. [/ Латекс]

Решение для (b)

Здесь угловая скорость уменьшается от [latex] \ boldsymbol {26.2}} \\ {} & \ boldsymbol {=} & \ boldsymbol {0.300 \ textbf {s.}} \ End {array} [/ latex]

Обсуждение

Обратите внимание, что угловое ускорение, когда девушка вращает колесо, небольшое и положительное; для получения заметной угловой скорости требуется 5 с. Когда она нажимает на тормоз, угловое ускорение велико и отрицательно. Угловая скорость быстро стремится к нулю. В обоих случаях отношения аналогичны тому, что происходит с линейным движением. Например, когда вы врезаетесь в кирпичную стену, происходит сильное замедление — изменение скорости велико за короткий промежуток времени.

Если бы велосипед в предыдущем примере был на колесах, а не перевернут, он сначала разогнался бы по земле, а затем остановился бы. Эту связь между круговым движением и линейным движением необходимо исследовать. Например, было бы полезно знать, как связаны линейное и угловое ускорение. При круговом движении линейное ускорение составляет по касательной к окружности в интересующей точке, как показано на рисунке 2. Таким образом, линейное ускорение называется касательным ускорением [latex] \ boldsymbol {a _ {\ textbf {t}}}.[/ латекс]

Рис. 2. При круговом движении линейное ускорение a возникает при изменении величины скорости: a касается движения. В контексте кругового движения линейное ускорение также называется тангенциальным ускорением a t .

Линейное или тангенциальное ускорение относится к изменениям величины скорости, но не ее направления. Из главы 6 мы знаем, что при круговом движении центростремительное ускорение [latex] \ boldsymbol {a _ {\ textbf {c}}}, [/ latex] относится к изменениям направления скорости, но не ее величине. .Объект, совершающий круговое движение, испытывает центростремительное ускорение, как показано на рисунке 3. Таким образом, [latex] \ boldsymbol {a _ {\ textbf {t}}} [/ latex] и [latex] \ boldsymbol {a _ {\ textbf {c} }} [/ latex] перпендикулярны и независимы друг от друга. Касательное ускорение [латекс] \ boldsymbol {a _ {\ textbf {t}}} [/ latex] напрямую связано с угловым ускорением [latex] \ boldsymbol {\ alpha} [/ latex] и связано с увеличением или уменьшением скорость, но не ее направление.

Рис. 3. Центростремительное ускорение a c возникает при изменении направления скорости; он перпендикулярен круговому движению.Таким образом, центростремительное и тангенциальное ускорения перпендикулярны друг другу.

Теперь мы можем найти точную связь между линейным ускорением [latex] \ boldsymbol {a _ {\ textbf {t}}} [/ latex] и угловым ускорением [latex] \ boldsymbol {\ alpha}. [/ Latex] Потому что линейное ускорение пропорциональна изменению величины скорости, она определена (как и в главе 2 «Одномерная кинематика») равной

.

[латекс] \ boldsymbol {a _ {\ textbf {t}} \: =} [/ latex] [латекс] \ boldsymbol {\ frac {\ Delta {v}} {\ Delta {t}}.} [/ латекс]

Для кругового движения обратите внимание, что [latex] \ boldsymbol {v = r \ omega}, [/ latex], так что

[латекс] \ boldsymbol {a _ {\ textbf {t}} \: =} [/ latex] [латекс] \ boldsymbol {\ frac {\ Delta (r \ omega)} {\ Delta {t}}.} [ / латекс]

Радиус [латекс] \ boldsymbol {r} [/ latex] постоянен для кругового движения, поэтому [латекс] \ boldsymbol {\ Delta (r \ omega) = r (\ Delta \ omega)}. [/ Latex] Таким образом,

[латекс] \ boldsymbol {a _ {\ textbf {t}} = r} [/ latex] [латекс] \ boldsymbol {\ frac {\ Delta \ omega} {\ Delta {t}}.} [/ Латекс]

По определению [латекс] \ boldsymbol {\ alpha = \ frac {\ Delta \ omega} {\ Delta {t}}}.[/ latex] Таким образом,

[латекс] \ boldsymbol {a _ {\ textbf {t}} = r \ alpha}, [/ latex]

или

[латекс] \ boldsymbol {\ alpha \: =} [/ latex] [латекс] \ boldsymbol {\ frac {a _ {\ textbf {t}}} {r}.} [/ Latex]

Эти уравнения означают, что линейное ускорение и угловое ускорение прямо пропорциональны. Чем больше угловое ускорение, тем больше линейное (тангенциальное) ускорение, и наоборот. Например, чем больше угловое ускорение ведущих колес автомобиля, тем больше ускорение автомобиля.Радиус тоже имеет значение. Например, чем меньше колесо, тем меньше его линейное ускорение для данного углового ускорения [латекс] \ boldsymbol {\ alpha}. [/ Latex]

Пример 2: Расчет углового ускорения колеса мотоцикла

Мощный мотоцикл может разогнаться от 0 до 30,0 м / с (около 108 км / ч) за 4,20 с. Каково угловое ускорение его колес радиусом 0,320 м? (См. Рисунок 4.)

Рисунок 4 . Линейное ускорение мотоцикла сопровождается угловым ускорением его колес.

Стратегия

Нам дана информация о линейных скоростях мотоцикла. Таким образом, мы можем найти его линейное ускорение [latex] \ boldsymbol {a _ {\ textbf {t}}}. [/ Latex] Тогда выражение [latex] \ boldsymbol {\ alpha = \ frac {a _ {\ textbf {t }}} {r}} [/ latex] можно использовать для определения углового ускорения.

Решение

Линейное ускорение

[латекс] \ begin {array} {lcl} \ boldsymbol {a _ {\ textbf {t}}} & \ boldsymbol {=} & \ boldsymbol {\ frac {\ Delta {v}} {\ Delta {t}} } \\ {} & \ boldsymbol {=} & \ boldsymbol {\ frac {30.2.} \ end {array} [/ latex]

Мы также знаем радиус колес. Ввод значений для [latex] \ boldsymbol {a _ {\ textbf {t}}} [/ latex] и [latex] \ boldsymbol {r} [/ latex] в [latex] \ boldsymbol {\ alpha = \ frac {a_ {\ textbf {t}}} {r}}, [/ latex] получаем

[латекс] \ begin {array} {lcl} \ boldsymbol {\ alpha} & \ boldsymbol {=} & \ boldsymbol {\ frac {a _ {\ textbf {t}}} {r}} \\ {} & \ boldsymbol {=} & \ boldsymbol {\ frac {7.14 \ textbf {m / s}} {20.320 \ textbf {m}}} \\ {} & \ boldsymbol {=} & \ boldsymbol {22.2.} \ end {array} [/ latex]

Обсуждение

Радианы безразмерны и присутствуют в любом соотношении между угловыми и линейными величинами.

До сих пор мы определили три вращательные величины — [латекс] \ boldsymbol {\ theta, \: \ omega}, [/ latex] и [латекс] \ boldsymbol {\ alpha}. [/ Latex] Эти величины аналогичны трансляционные величины [латекс] \ boldsymbol {x}, \: \ boldsymbol {v}, [/ latex] и [latex] \ boldsymbol {a}. [/ latex] В таблице 1 показаны вращательные величины, аналогичные трансляционные величины и отношения между ними.

Вращающийся Трансляционный Отношения
[латекс] \ boldsymbol {\ theta} [/ латекс] [латекс] \ boldsymbol {x} [/ латекс] [латекс] \ boldsymbol {\ theta = \ frac {x} {r}} [/ латекс]
[латекс] \ boldsymbol {\ omega} [/ латекс] [латекс] \ boldsymbol {v} [/ латекс] [латекс] \ boldsymbol {\ omega = \ frac {v} {r}} [/ латекс]
[латекс] \ boldsymbol {\ alpha} [/ латекс] [латекс] \ boldsymbol {a} [/ латекс] [латекс] \ boldsymbol {\ alpha = \ frac {a _ {\ textbf {t}}} {r}} [/ latex]
Таблица 1. Вращательные и поступательные величины.

УСТАНОВКА ПОДКЛЮЧЕНИЙ: ЭКСПЕРИМЕНТ НА ​​ДОМУ


Сядьте, поставив ноги на землю, на вращающийся стул. Поднимите одну ногу так, чтобы она была разогнута (выпрямлена). Используя другую ногу, начните вращаться, отталкиваясь от земли. Прекратите толкать землю ногой, но позвольте стулу вращаться. От исходной точки, с которой вы начали, нарисуйте угол, угловую скорость и угловое ускорение вашей ноги как функцию времени в виде трех отдельных графиков.Оцените величину этих величин.

Проверьте свое понимание

1: Угловое ускорение — это вектор, имеющий как величину, так и направление. Как обозначить его величину и направление? Проиллюстрируйте на примере.

PHET EXPLORATIONS: LADYBUG REVOLUTION

Присоединяйтесь к божьей коровке и исследуйте вращательное движение. Вращайте карусель, чтобы изменить ее угол, или выберите постоянную угловую скорость или угловое ускорение.Изучите, как круговое движение связано с координатами x, y, скоростью и ускорением жука, используя векторы или графики.

Рисунок 5. Божья коровка Revolution
  • Равномерное круговое движение — это движение с постоянной угловой скоростью [латекс] \ boldsymbol {\ omega = \ frac {\ Delta \ theta} {\ Delta {t}}}. [/ Latex]
  • При неравномерном круговом движении скорость изменяется со временем, а скорость изменения угловой скорости (т.е. угловое ускорение) равна [latex] \ boldsymbol {\ alpha = \ frac {\ Delta \ omega} {\ Delta {t} }}.[/ латекс]
  • Линейное или тангенциальное ускорение относится к изменениям величины скорости, но не ее направления, заданному как [latex] \ boldsymbol {a _ {\ textbf {t}} = \ frac {\ Delta {v}} {\ Delta {t} }}. [/ latex]
  • Для кругового движения обратите внимание, что [latex] \ boldsymbol {v = r \ omega}, [/ latex] так, чтобы

    [латекс] \ boldsymbol {a _ {\ textbf {t}} \: =} [/ latex] [латекс] \ boldsymbol {\ frac {\ Delta (r \ omega)} {\ Delta {t}}}. [ / латекс]

  • Радиус r постоянен для кругового движения, поэтому [латекс] \ boldsymbol {\ Delta (r \ omega) = r \ Delta \ omega}.[/ latex] Таким образом,

    [латекс] \ boldsymbol {a _ {\ textbf {t}} = r} [/ latex] [латекс] \ boldsymbol {\ frac {\ Delta \ omega} {\ Delta {t}}}. [/ Latex]

  • По определению [латекс] \ boldsymbol {\ Delta \ omega / \ Delta {t} = \ alpha}. [/ Latex] Таким образом,

    [латекс] \ boldsymbol {a _ {\ textbf {t}} = r \ alpha} [/ latex]

    или

    [латекс] \ boldsymbol {\ alpha =} [/ latex] [латекс] \ boldsymbol {\ frac {a _ {\ textbf {t}}} {r}}. [/ Latex]

Концептуальные вопросы

1: Между вращательными и поступательными физическими величинами существуют аналогии.Определите вращательный член, аналогичный каждому из следующих: ускорение, сила, масса, работа, поступательная кинетическая энергия, линейный импульс, импульс.

2: Объясните, почему центростремительное ускорение изменяет направление скорости при круговом движении, но не ее величину.

3: При круговом движении тангенциальное ускорение может изменять величину скорости, но не ее направление. Поясните свой ответ.

4: Предположим, что кусок пищи стоит на краю вращающейся пластины микроволновой печи.Испытывает ли она ненулевое тангенциальное ускорение, центростремительное ускорение или и то, и другое, когда: а) пластина начинает вращаться? (б) Пластина вращается с постоянной угловой скоростью? (c) Пластина замедляется до полной остановки?

Задачи и упражнения

1: На пике торнадо имеет диаметр 60,0 м и скорость ветра 500 км / ч. Какова его угловая скорость в оборотах в секунду?

2: интегрированные концепции

Ультрацентрифуга ускоряется от состояния покоя до 100000 об / мин за 2 секунды.2} [/ latex] и кратные [latex] \ boldsymbol {g} [/ latex] этой точки на полных оборотах?

3: интегрированные концепции

У вас есть точильный камень (диск) весом 90,0 кг, радиусом 0,340 м и вращающимся со скоростью 90,0 об / мин, и вы прижимаете к нему стальной топор с радиальной силой 20,0 Н. 2}.[/ latex] Направление углового ускорения вдоль фиксированной оси обозначается знаком + или a -, так же как направление линейного ускорения в одном измерении обозначается знаком + или a -. Например, представьте, что гимнастка делает сальто вперед. Ее угловой момент был бы параллелен ковру слева от нее. Величина ее углового ускорения будет пропорциональна ее угловой скорости (скорости вращения) и ее моменту инерции относительно оси вращения.

Задачи и упражнения

1:

[латекс] \ boldsymbol {\ omega = 0.2} [/ латекс]

(b) [латекс] \ boldsymbol {27 \ textbf {rev}} [/ латекс]

.

alexxlab / 15.01.2021 / Разное

Добавить комментарий

Почта не будет опубликована / Обязательны для заполнения *